Is there any good reason to write “it is easy to see”? [closed]












69















Many academic papers, particularly in mathematics and similar fields, use the phrase "it is easy to see that..." (e.g. in a mathematical proof). I never understood why this sentence is used. Such a sentence is inaccurate at best, since it is not easy for everyone to see; maybe it is easy for the author or some of the readers, but there are certainly readers to whom it is not easy.
Authors often try to shorten their paper as much as possible, so it is not clear why they would lengthen their paper by an inaccurate sentence.



Is there any good reason for an author to use this phrase?










share|improve this question















closed as primarily opinion-based by Stephan Kolassa, corey979, Morgan Rodgers, Dmitry Grigoryev, Richard Erickson May 17 at 15:40


Many good questions generate some degree of opinion based on expert experience, but answers to this question will tend to be almost entirely based on opinions, rather than facts, references, or specific expertise. If this question can be reworded to fit the rules in the help center, please edit the question.



















  • Comments are not for extended discussion; this conversation has been moved to chat.

    – eykanal
    May 14 at 17:41






  • 3





    The answer to your question is "no." There is a paragraph in Halberstam and Roth's Sequences which begins, "it is clear that...." then all holy hell breaks loose. I spent a month trying to get through that paragraph and then gave up and just continued reading. It turns out that the paragraph ends ".... is far from obvious." That was the end of additive number theory for me. 40 years later, I'm still mad. "It is clear that this stuff is far from obvious." Sheesh.

    – B. Goddard
    May 14 at 23:34
















69















Many academic papers, particularly in mathematics and similar fields, use the phrase "it is easy to see that..." (e.g. in a mathematical proof). I never understood why this sentence is used. Such a sentence is inaccurate at best, since it is not easy for everyone to see; maybe it is easy for the author or some of the readers, but there are certainly readers to whom it is not easy.
Authors often try to shorten their paper as much as possible, so it is not clear why they would lengthen their paper by an inaccurate sentence.



Is there any good reason for an author to use this phrase?










share|improve this question















closed as primarily opinion-based by Stephan Kolassa, corey979, Morgan Rodgers, Dmitry Grigoryev, Richard Erickson May 17 at 15:40


Many good questions generate some degree of opinion based on expert experience, but answers to this question will tend to be almost entirely based on opinions, rather than facts, references, or specific expertise. If this question can be reworded to fit the rules in the help center, please edit the question.



















  • Comments are not for extended discussion; this conversation has been moved to chat.

    – eykanal
    May 14 at 17:41






  • 3





    The answer to your question is "no." There is a paragraph in Halberstam and Roth's Sequences which begins, "it is clear that...." then all holy hell breaks loose. I spent a month trying to get through that paragraph and then gave up and just continued reading. It turns out that the paragraph ends ".... is far from obvious." That was the end of additive number theory for me. 40 years later, I'm still mad. "It is clear that this stuff is far from obvious." Sheesh.

    – B. Goddard
    May 14 at 23:34














69












69








69


7






Many academic papers, particularly in mathematics and similar fields, use the phrase "it is easy to see that..." (e.g. in a mathematical proof). I never understood why this sentence is used. Such a sentence is inaccurate at best, since it is not easy for everyone to see; maybe it is easy for the author or some of the readers, but there are certainly readers to whom it is not easy.
Authors often try to shorten their paper as much as possible, so it is not clear why they would lengthen their paper by an inaccurate sentence.



Is there any good reason for an author to use this phrase?










share|improve this question
















Many academic papers, particularly in mathematics and similar fields, use the phrase "it is easy to see that..." (e.g. in a mathematical proof). I never understood why this sentence is used. Such a sentence is inaccurate at best, since it is not easy for everyone to see; maybe it is easy for the author or some of the readers, but there are certainly readers to whom it is not easy.
Authors often try to shorten their paper as much as possible, so it is not clear why they would lengthen their paper by an inaccurate sentence.



Is there any good reason for an author to use this phrase?







writing-style






share|improve this question















share|improve this question













share|improve this question




share|improve this question








edited May 12 at 21:21









Andrew Morton

1033




1033










asked May 12 at 12:26









Erel Segal-HaleviErel Segal-Halevi

7,045104376




7,045104376




closed as primarily opinion-based by Stephan Kolassa, corey979, Morgan Rodgers, Dmitry Grigoryev, Richard Erickson May 17 at 15:40


Many good questions generate some degree of opinion based on expert experience, but answers to this question will tend to be almost entirely based on opinions, rather than facts, references, or specific expertise. If this question can be reworded to fit the rules in the help center, please edit the question.









closed as primarily opinion-based by Stephan Kolassa, corey979, Morgan Rodgers, Dmitry Grigoryev, Richard Erickson May 17 at 15:40


Many good questions generate some degree of opinion based on expert experience, but answers to this question will tend to be almost entirely based on opinions, rather than facts, references, or specific expertise. If this question can be reworded to fit the rules in the help center, please edit the question.















  • Comments are not for extended discussion; this conversation has been moved to chat.

    – eykanal
    May 14 at 17:41






  • 3





    The answer to your question is "no." There is a paragraph in Halberstam and Roth's Sequences which begins, "it is clear that...." then all holy hell breaks loose. I spent a month trying to get through that paragraph and then gave up and just continued reading. It turns out that the paragraph ends ".... is far from obvious." That was the end of additive number theory for me. 40 years later, I'm still mad. "It is clear that this stuff is far from obvious." Sheesh.

    – B. Goddard
    May 14 at 23:34



















  • Comments are not for extended discussion; this conversation has been moved to chat.

    – eykanal
    May 14 at 17:41






  • 3





    The answer to your question is "no." There is a paragraph in Halberstam and Roth's Sequences which begins, "it is clear that...." then all holy hell breaks loose. I spent a month trying to get through that paragraph and then gave up and just continued reading. It turns out that the paragraph ends ".... is far from obvious." That was the end of additive number theory for me. 40 years later, I'm still mad. "It is clear that this stuff is far from obvious." Sheesh.

    – B. Goddard
    May 14 at 23:34

















Comments are not for extended discussion; this conversation has been moved to chat.

– eykanal
May 14 at 17:41





Comments are not for extended discussion; this conversation has been moved to chat.

– eykanal
May 14 at 17:41




3




3





The answer to your question is "no." There is a paragraph in Halberstam and Roth's Sequences which begins, "it is clear that...." then all holy hell breaks loose. I spent a month trying to get through that paragraph and then gave up and just continued reading. It turns out that the paragraph ends ".... is far from obvious." That was the end of additive number theory for me. 40 years later, I'm still mad. "It is clear that this stuff is far from obvious." Sheesh.

– B. Goddard
May 14 at 23:34





The answer to your question is "no." There is a paragraph in Halberstam and Roth's Sequences which begins, "it is clear that...." then all holy hell breaks loose. I spent a month trying to get through that paragraph and then gave up and just continued reading. It turns out that the paragraph ends ".... is far from obvious." That was the end of additive number theory for me. 40 years later, I'm still mad. "It is clear that this stuff is far from obvious." Sheesh.

– B. Goddard
May 14 at 23:34










12 Answers
12






active

oldest

votes


















81














Mathematical papers, especially, and many others, are not written for the complete novice with no understanding of the field. One tries to tailor the explanation of a new concept to the general level of understanding of the audience and tries to avoid being overly pedantic giving every detail and argument back to Euclid.



College lecturers often use such phrases, knowing the level that their students should have attained and, thus, permitting the lecture to flow more smoothly and be less boring.



But writers do the same thing, envisioning the reader of the paper, or perhaps just the reviewers. If I say such a thing and the reviewer calls me on it, then I know I have to say more. But I trust that the reviewer can validly stand in place of the audience and will be able to easily fill such gaps.



When such a phrase is used (we can conclude X) it implies that X flows from the previous statement(s) and isn't just an unsupported statement inserted into the flow. It indicates a hopefully simple, but unstated, argument.



The alternative of including the complete argument is longer, more pedantic, more boring papers.



The other alternative of starting every sentence with "Therefore,..." is also stilted and, eventually, boring. Some variation of phrasing makes the reading more pleasant. Writers, in general, like to have a repertoire of, more or less, equivalent phrases to help the flow.



On the other hand, it has happened that the statement is used incorrectly when it takes a lot of argument and deep insight to go from point A to point B. Sometimes the author just doesn't notice the width of the gap. But, for the most part, it seems to work out.






share|improve this answer





















  • 76





    One wants to indicate that X flows from the previous and isn't just an unsupported statement inserted into the flow. The alternative (longer) is to include the complete argument.

    – Buffy
    May 12 at 12:47








  • 41





    +1 for and isn't just an unsupported statement inserted into the flow. One of the things I used to find so frustrating when reading physics texts (and worse yet, engineering, chemistry, etc.; in the case of the many social science texts I had to read for required "elective courses", it was actually not much of a concern, since pretty much everything in those texts was clearly non-deductive) was in trying to determine which author assertions were statements whose validity I was supposed to be able to determine based on prior material in the text (continued)

    – Dave L Renfro
    May 12 at 13:22








  • 37





    and which author assertions were simply assertions of the FYI variety. Regarding your answer, besides alerting the reader to the fact that an assertion is intended to be deducible by the reader, saying "it is easy to see that" (when used correctly) assists the reader in what to consider and how deep to go. For an analogy, I used to play chess a lot when I was young (mostly before high school), and it was MUCH easier to win over a weaker opponent if I KNEW the opponent was weak, because then I could go for higher risk moves that lead to quick capture of pieces.

    – Dave L Renfro
    May 12 at 13:31








  • 19





    Yes, +1 for distinguishing it from an unsupported - or even apparently unsupported - statement. In a math paper, if I come across what appears to be an unsupported statement in the flow of a proof, a common explanation is that it's something that was shown earlier in the paper that I've forgotten about. "It is easy to see" stops me going back and looking for it.

    – Especially Lime
    May 12 at 15:01








  • 13





    @ErelSegal-Halevi "It is easy to see X" is code for "I am skipping some steps, but X follows." I very much appreciate if authors let me know when they skip a few steps so that I won't try to find them in the text.

    – Jair Taylor
    May 12 at 20:34



















63














Several times during seminars I heard the following exchange, which sounds like a joke but actually isn't:



Audience member: Why is X true?



Speaker: Oh, it's obvious.



Audience member: OK, thanks! [sits down satisfied]





Especially in mathematics, it is often helpful to let the Reader know what level of difficulty and/or complexity each step is. The approach you (or at least I) would take differs significantly depending on this.



If a step is supposed to be easy and it's not quite my field, I'd take note that this sort of transition is standard in these parts and move on. Conversely, if the step is supposed to be easy and it is my field and it doesn't seem easy at all, then it's a strong indication that either the author is wrong (rare) or that my understanding is insufficient and I should think about this transition until it seems easy (not just until I find any way to justify it).



If a step is supposed to be moderate in difficulty, I might or might not work out the details myslef as an exercise, but I will also be aware that some amount of work goes into making it possible - helpful when evaluating if an argument is plausible and also in identifying where the content of the paper really is (especially in papers that are heavy on definitions, it's often a nontrivial task to figure out which transitions are easy but unfamiliar and which are responsible for the actual progress).



Finally, if a step is said to be difficult then there obviously should be a reference or a proof. If it's a proof, then more likely than not this is the point where the progress is being made in the paper, so if I'm reading the paper this is the part I would study in the most detail. If it's a reference, I would make a mental note that it's a potentially strong tool to keep in my arsenal - also, I would know better than to attempt to reproduce the result myself.



Note that it's not always all that easy to determine which is which without the Author explicitly making a judgement. The short phrase "By Thm. C in [42] we have X" could expand into either of "It is easy to see that X (see Thm C in [42] for details)" and "Because of the deep theorem of Smith (Thm C in [42]) we have X".





Having said all of the above, I want to add that personally, I dislike the phrase "It is easy to see". I understand the sentiment, but if the paper is read by anyone other than the experts in the field (maybe undergrads or experts in another field) then chances are it's not going to be easy to see to all the Readers, and the Readers who don't find it easy might feel bad about it in one way or another. If I'm already taking the time to explain myself for not explaining a transition I try to give some more details: either a super-short sketch of a proof, or a phrase like "It follows by an application of standard techniques that ..." or "A simple but mundane computation shows that...", etc.






share|improve this answer



















  • 3





    +1 for "Especially in mathematics, it is often helpful to let the Reader know what level of difficulty and/or complexity each step is." Sometimes it's difficult to know if the leap from one step to another is difficult or not. By saying "It's easy to see" you're letting the reader know that probably the first/simplest line of reasoning they attempt to bridge the gap is the correct one, and you therefore help the reader not waste too much mental energy in trying to determine the soundness of their ad-hoc reasoning, since it is most likely correct.

    – ChocolateAndCheese
    May 14 at 0:33






  • 1





    Just to clarify, the "OK, thanks" in the exchange does not mean "I get it", but more than "I trust you that you are right". When the audience does not get a thing, it's also natural to wonder if the speaker is really sure about that. Therefore, the beginning question "Why is X true?" really means "Are you sure that X is true?"

    – Ooker
    May 14 at 8:36






  • 1





    @Ooker - It's not so much about doubting the speaker (after all, they are the expert on the subject and have presumably spent some time preparing the talk and making sure that what they say is correct). It's more about the desire to understand why X is true. What the speaker is effectively saying is "I assure you that this transition is correct and that if you had all the relevant ingredients at hand and maybe spent 5 minutes with a piece of paper, you would easily reproduce it; there's nothing deep or particularly interesting happening here.".

    – Jakub Konieczny
    May 14 at 10:23






  • 2





    @Ooker It can even mean "I get it" (if preceeded by 5sec pausing). In informal discussions with colleagues I certainly had it happen that upon being told that something was obvious I could immediately see that it was, indeed, obvious, which had not been obvious to me before I asked.

    – Arno
    May 14 at 10:38






  • 3





    @Arno - That's a very fair point. As a slight variation on the same theme, there are often situations there there is a very obvious way to try and prove something, with the caveat that at each step something could potentially go wrong and become difficult. In a case like that, "It's obvious" might mean: The obvious argument works, nothing breaks down along the way.

    – Jakub Konieczny
    May 14 at 10:45



















15














There are no good reasons for phrases like that one. If something is "easy to see" or is "obvious", then there is no point in emphasizing such truism.



However, if no additional knowledge in needed and the following steps are quite straightforward, then one can use the advice form the Mathematical writing article in The Princeton companion to applied mathematics:




The question of how much detail to give is related to the question of
how formal to be, but it is not the same question. It is true that
there is a tendency in informal mathematical writing to leave out
details, but with even the most formal writing a decision has to be
made about how much detail to give; it is just that in formal writing
one probably wants to signal more carefully when details have been
left out. This can be done in various ways. One can use expressions
such as “It is an easy exercise to check that...,” or “The second case
is similar,” which basically say to the reader, “I have decided not to
spell out this part of the argument.” One can also give small hints,
such as “By compactness,” or “An obvious inductive argument now
shows that...,” or “Interchanging the order of summation and
simplifying, we obtain....”



If you do decide to leave out detail, it
is a good idea to signal to the reader how difficult it would be to
put that detail in. A mistake that some writers make is to give
references to other papers for arguments that can easily be worked
out by the reader, without saying that the particular result that is
needed is easy. This is straight- forwardly misleading; it suggests
that the best thing to do is to go and look up the other paper when in
fact the best thing to do is to work out the argument for oneself.




Also from How to read and understand a paper in the same book:




In mathematical writing certain standard phrases are used that have
particular meanings. “It follows that” or “it is easy to see that”
mean that the next statement can be proved without using any new ideas

and that giving the details would clutter the text. The detail may,
however, be tedious. The shorter “hence,” “therefore,” or “so” imply a
more straightforward conclusion. “It can be shown that” again implies
that details are not felt to be worth including but is noncommittal
about the difficulty of the proof.







share|improve this answer





















  • 1





    From a strictly grammatical standpoint, yes, you're correct. But humans often need nudges to connect ideas mentally. It lowers the cognition required to process the more important details at hand.

    – Carl V. Lewis
    May 12 at 20:59






  • 3





    Such phrases are not pointless, because they provide metadata. Language gives us not only the ability to transfer information to others, and also information about that information, and even information about the information about that information. It's very common to use language to convey meaning on multiple levels at once.

    – barbecue
    May 12 at 21:03






  • 3





    @CarlV.Lewis This is exactly why it is better give more useful breadcrumbs to connect ideas, e.g.: "From the application of Theorem 4.1 to system 2.4, it is easy to see..."

    – homocomputeris
    May 12 at 22:46






  • 10





    "There are no good reasons..." ... proceeding several good reasons. ;)

    – AnoE
    May 13 at 16:07






  • 3





    If something is "easy to see" but I don't see it, that is a very useful cue for me to go back and check if I missed something simple, rather than trying more complicated methods of deriving the "obvious" result. A simple line of reasoning is easier to deduce if you can exclude the more complicated ones. Even if X is obvious to most people, it will be obvious to even more people if you label it as an "obvious result".

    – Nuclear Wang
    May 13 at 19:05



















13














One common way that I interpret "It is easy to see X" is as an ever so slight shortening of "it easily follows that X" (which I prefer). This itself is a shortened version of "The preceding assertion implies X", or perhaps a pluralized version such as "The preceding assertions (or definitions, or assertions and definitions, or...) imply X".



There is information contained here, namely a logical implication, that would be lost if one simply wrote "X".






share|improve this answer































    10














    "It is easy to see" should be used sparingly. If one can say how it is easy to see why the statement is true, in a similar number of words, then this strategy is prefered. Alternative, but more specific expressions, than "it is easy to see" include




    • After routine algebra, ...

    • By theorem X, ...

    • By definition of Y, ...


    Note the phrase "It is easy to see that" is actually longer than all of the above.



    Sometimes the reason why it is easy to see something cannot be explained in so few words. In which case, if it is truly easy to see for the target audience, using this phrase is useful to signal to the reader that if they don't follow, its probably some trivial thing that they've messed up in their head, rather than a need to think deeply about the statement. But even then, the phrase should only be used when it can't be made more specific.






    share|improve this answer



















    • 3





      Your suggestion to include short specific suggestive methods of proof is something I began realizing many years ago had little effect on verbal length or content tangency while greatly helping some readers, and since then I've tried to do this as much as is reasonable, often adding such explanatory phrases on the second or third rewrite, as I don't always think about it the first time around. Other examples: Instead of "by an easy integration", say "letting u = ..." or "using integration by parts". Instead of "it is easy to show convergence", say "by limit comparison with a geometric series".

      – Dave L Renfro
      May 14 at 10:24











    • @DaveLRenfro Agreed, but at least "by easy integration" is better than easy to see. It at least signals to the reader that, awe I need to compute this integral to get to the next statement, the more specific the better assuming not many words are required. Absolutely agree.

      – WetlabStudent
      May 15 at 3:50



















    9














    My personal opinion is that "it is easy to see" should really mean that it is easy for a reader who is the target audience to see, and can be used when that part of the proof is not an important piece, such as if it is a routine or tedious but uninteresting calculation. For example, it is better to write "it is easy to see that algorithm A runs in O(n^3) time" if it is just a bunch of for-loops that obviously take O(n^3) time, than to give a long and tedious proof just for the sake of formal rigour.



    That said, it does happen that sometimes what one thinks is "easy to see" is not so easy for others to see, or perhaps even oneself after a few months of not looking at it. Hence having reviewers helps.






    share|improve this answer































      8














      Papers are not written with exhaustive detail. There is always a balance between explicitly stating your reasoning and sticking to the topic at hand. Sometimes there are points that don't take great insight to understand, and would be easy (though perhaps lengthy) for the reader to derive on their own. This is the first function of "it is easy to see".



      Another aspect is managing tone and audience expectation. Papers are written for an expert audience with the premise of pushing the bleeding edge of human knowledge. If the author of a paper states something obvious or well known, the reader may be confused: If it is emphasized in a modern paper, could it be that the author is not just a fool restating things everybody knows, but actually means to state something different? The competent reader could be confused by what the author meant to accomplish in stating something already well known or "easy to see". So the author preempts this, by acknowledging the point as something well-known and not novel, but mentioned for clarity and/or as a reminder, and not something to be taken as a novel or interesting claim. This is the second function of "it is easy to see".



      Some people like to criticize statements like "it can be shown" ("then why don't you show it?" - because it would be a distracting digression) or "it's obvious" ("then why say it?" - to show that the author is aware of it). They assume it is an exercise in arrogance on the part of the author. However, written communication is not just the text. There is also subtext and context. Well-structured text has a central point and a coherent tone. Going on every possible tangent, digressing into topics of wildly different level, does not necessarily serve these qualities. As such, phrases like "it is easy to see" can serve an important point in enabling effective communication of novel findings to an advanced, technical audience that has less time available to read than there are papers of interest.






      share|improve this answer































        6














        Yes. It specifies a particular level of difficulty (not too easy and not too hard), thus managing the reader's expectations and directing their focus.



        The phrase will not be used for completely trivial deductions that can be done in half a second. If the previous sentence concluded that 2x=2, nobody would write "it is therefore easy to see that x=1". It's easy to see that it's easy to see, so there's no value in pointing out that it's easy to see.



        Likewise, if the deduction is difficult and requires hours to figure out, nobody would write "it's easy to see", because it's not, and saying it is will confuse the reader.



        It's the middle ground where the deduction can take a few seconds or maybe a few minutes, where the phrase is useful. A priori, the reader does not know the difficulty of the deduction - is this a half-second thing and I'm too dense to figure it out? Is this a difficult deduction and the author is remiss for neglecting a proof, or maybe he's just making stuff up?



        The statement "It's easy to see" signals that we're in the middle ground - No, you're not stupid for failing to recognize this immediately (if it were that easy, I wouldn't say anything). But yes, I'm confident that if you spend a minute you'll figure it out, so there's no need to encumber the paper with all the details.



        So much power in such a simple phrase.






        share|improve this answer

































          6














          The author most often writes this to justify that he does not include a proof, that you can do yourself in a reasonable time. This assumes that you have the basic knowlege that is required in your field to understand the average paper.



          If you think you're missing some parts and cannot see why it is easy, you usually can look up the "easy to see" (after eight pages of calculations) proofs in other manuscripts, text books or in the internet.



          If they would include this proofs, the reviewers and editors would ask them, why they proof something, that is known from text books or "easy" to see. A paper should contribute something new and should not (and cannot) proof any details that it builds up on.



          For more stuff that is not well-known or more complicated, the authors should provide a citation and write "the derivation of the formula and the proof can be found in [42]".



          But when you cite a text book, then people can ask why do you cite text book A and not text book B? The correct citation would be the original paper, but it will usually be much harder to understand the topic in the original paper than in a text book.



          And there are of course some hand-waving, when the author knows something to be true, but would need more time to proof it himself and does not consider it to be worth the effort.






          share|improve this answer

































            1















            "You'll notice that ...."




            First person always reads smoother, commands attention, and prevents overwtiting.



            That said, you'll want to be clear who your audience is at the outset of the publication. State it explicitly who should be reading the text, and the assumed knowledge level of the subject matter.






            share|improve this answer































              0














              Yes, though it is sometimes abused.



              It's an assertion that if you think something is true you've probably got it right. It also sets the tone of what you're about to have to think through. It's common in other walks of life the other way round: if you have something that you are asserting that is non-trivial and is going to take some working though, you give your audience a warning.



              In maths, perhaps arrogantly (though unfortunately in my experience accurately), it is assumed that the statements in papers, talks, books, etc are involved. Thus instead of caveating each sentence with "this is hard", the few that are easy are caveated instead (simply as a way to reduce verbosity).



              The problem comes when the caveat is incorrect or is only correct after something highly non-trivial has become internalised.
              In this case it alienates those readers. As I imagine you can guess it not only hampers progress, it can also be seen as a bit of a middle finger. "If you don't get this you must not be cut out to be doing this sort of thing". Perhaps in some cases this is how its meant or just laziness. However its not always easy to know what will help your readers most and "nothing deep happens here" is far more likely.



              I don't like using "trivial" or "clear" because of the potential to be misread, but I have often wanted to express that sentiment.






              share|improve this answer































                0














                Yes, there are good reasons for the general meaning of phrases like this, but "It is easy to see that..." is a very poor choice of phrase for this meaning. Others have already suggested better phrases.



                As Buffy expresses, it's not expected for advanced papers to reference, let alone prove every conclusion used. The problem here is the passive voice, and, more importantly:



                It is easy to see. What is easy to see?



                I see two general phrase choices:




                1. Longer. If you want sentence flow to present a long thought, "From Y, we observe that with/from [few steps or concepts], X.", which carries the meaning more explicitly.


                2. Shorter. What is being said is, "Y. Y => X." (Y being the conclusion the reader is expected to know, from the audience the text is written for.)



                This way, you are at least specifying the subject of your sentence.



                Now, if Y is something you learn in high school, you would look silly specifying it; you would look as if you're proud that you can still do high school math. You write papers for your peers.



                It's a safe bet that one or more of your peers won't agree Y is obvious, but perhaps will say nothing in review to not "look dumb", when in fact s/he is your peer, just working in a different field.



                Widening the audience slightly from that seems a reasonable place to be.






                share|improve this answer
































                  12 Answers
                  12






                  active

                  oldest

                  votes








                  12 Answers
                  12






                  active

                  oldest

                  votes









                  active

                  oldest

                  votes






                  active

                  oldest

                  votes









                  81














                  Mathematical papers, especially, and many others, are not written for the complete novice with no understanding of the field. One tries to tailor the explanation of a new concept to the general level of understanding of the audience and tries to avoid being overly pedantic giving every detail and argument back to Euclid.



                  College lecturers often use such phrases, knowing the level that their students should have attained and, thus, permitting the lecture to flow more smoothly and be less boring.



                  But writers do the same thing, envisioning the reader of the paper, or perhaps just the reviewers. If I say such a thing and the reviewer calls me on it, then I know I have to say more. But I trust that the reviewer can validly stand in place of the audience and will be able to easily fill such gaps.



                  When such a phrase is used (we can conclude X) it implies that X flows from the previous statement(s) and isn't just an unsupported statement inserted into the flow. It indicates a hopefully simple, but unstated, argument.



                  The alternative of including the complete argument is longer, more pedantic, more boring papers.



                  The other alternative of starting every sentence with "Therefore,..." is also stilted and, eventually, boring. Some variation of phrasing makes the reading more pleasant. Writers, in general, like to have a repertoire of, more or less, equivalent phrases to help the flow.



                  On the other hand, it has happened that the statement is used incorrectly when it takes a lot of argument and deep insight to go from point A to point B. Sometimes the author just doesn't notice the width of the gap. But, for the most part, it seems to work out.






                  share|improve this answer





















                  • 76





                    One wants to indicate that X flows from the previous and isn't just an unsupported statement inserted into the flow. The alternative (longer) is to include the complete argument.

                    – Buffy
                    May 12 at 12:47








                  • 41





                    +1 for and isn't just an unsupported statement inserted into the flow. One of the things I used to find so frustrating when reading physics texts (and worse yet, engineering, chemistry, etc.; in the case of the many social science texts I had to read for required "elective courses", it was actually not much of a concern, since pretty much everything in those texts was clearly non-deductive) was in trying to determine which author assertions were statements whose validity I was supposed to be able to determine based on prior material in the text (continued)

                    – Dave L Renfro
                    May 12 at 13:22








                  • 37





                    and which author assertions were simply assertions of the FYI variety. Regarding your answer, besides alerting the reader to the fact that an assertion is intended to be deducible by the reader, saying "it is easy to see that" (when used correctly) assists the reader in what to consider and how deep to go. For an analogy, I used to play chess a lot when I was young (mostly before high school), and it was MUCH easier to win over a weaker opponent if I KNEW the opponent was weak, because then I could go for higher risk moves that lead to quick capture of pieces.

                    – Dave L Renfro
                    May 12 at 13:31








                  • 19





                    Yes, +1 for distinguishing it from an unsupported - or even apparently unsupported - statement. In a math paper, if I come across what appears to be an unsupported statement in the flow of a proof, a common explanation is that it's something that was shown earlier in the paper that I've forgotten about. "It is easy to see" stops me going back and looking for it.

                    – Especially Lime
                    May 12 at 15:01








                  • 13





                    @ErelSegal-Halevi "It is easy to see X" is code for "I am skipping some steps, but X follows." I very much appreciate if authors let me know when they skip a few steps so that I won't try to find them in the text.

                    – Jair Taylor
                    May 12 at 20:34
















                  81














                  Mathematical papers, especially, and many others, are not written for the complete novice with no understanding of the field. One tries to tailor the explanation of a new concept to the general level of understanding of the audience and tries to avoid being overly pedantic giving every detail and argument back to Euclid.



                  College lecturers often use such phrases, knowing the level that their students should have attained and, thus, permitting the lecture to flow more smoothly and be less boring.



                  But writers do the same thing, envisioning the reader of the paper, or perhaps just the reviewers. If I say such a thing and the reviewer calls me on it, then I know I have to say more. But I trust that the reviewer can validly stand in place of the audience and will be able to easily fill such gaps.



                  When such a phrase is used (we can conclude X) it implies that X flows from the previous statement(s) and isn't just an unsupported statement inserted into the flow. It indicates a hopefully simple, but unstated, argument.



                  The alternative of including the complete argument is longer, more pedantic, more boring papers.



                  The other alternative of starting every sentence with "Therefore,..." is also stilted and, eventually, boring. Some variation of phrasing makes the reading more pleasant. Writers, in general, like to have a repertoire of, more or less, equivalent phrases to help the flow.



                  On the other hand, it has happened that the statement is used incorrectly when it takes a lot of argument and deep insight to go from point A to point B. Sometimes the author just doesn't notice the width of the gap. But, for the most part, it seems to work out.






                  share|improve this answer





















                  • 76





                    One wants to indicate that X flows from the previous and isn't just an unsupported statement inserted into the flow. The alternative (longer) is to include the complete argument.

                    – Buffy
                    May 12 at 12:47








                  • 41





                    +1 for and isn't just an unsupported statement inserted into the flow. One of the things I used to find so frustrating when reading physics texts (and worse yet, engineering, chemistry, etc.; in the case of the many social science texts I had to read for required "elective courses", it was actually not much of a concern, since pretty much everything in those texts was clearly non-deductive) was in trying to determine which author assertions were statements whose validity I was supposed to be able to determine based on prior material in the text (continued)

                    – Dave L Renfro
                    May 12 at 13:22








                  • 37





                    and which author assertions were simply assertions of the FYI variety. Regarding your answer, besides alerting the reader to the fact that an assertion is intended to be deducible by the reader, saying "it is easy to see that" (when used correctly) assists the reader in what to consider and how deep to go. For an analogy, I used to play chess a lot when I was young (mostly before high school), and it was MUCH easier to win over a weaker opponent if I KNEW the opponent was weak, because then I could go for higher risk moves that lead to quick capture of pieces.

                    – Dave L Renfro
                    May 12 at 13:31








                  • 19





                    Yes, +1 for distinguishing it from an unsupported - or even apparently unsupported - statement. In a math paper, if I come across what appears to be an unsupported statement in the flow of a proof, a common explanation is that it's something that was shown earlier in the paper that I've forgotten about. "It is easy to see" stops me going back and looking for it.

                    – Especially Lime
                    May 12 at 15:01








                  • 13





                    @ErelSegal-Halevi "It is easy to see X" is code for "I am skipping some steps, but X follows." I very much appreciate if authors let me know when they skip a few steps so that I won't try to find them in the text.

                    – Jair Taylor
                    May 12 at 20:34














                  81












                  81








                  81







                  Mathematical papers, especially, and many others, are not written for the complete novice with no understanding of the field. One tries to tailor the explanation of a new concept to the general level of understanding of the audience and tries to avoid being overly pedantic giving every detail and argument back to Euclid.



                  College lecturers often use such phrases, knowing the level that their students should have attained and, thus, permitting the lecture to flow more smoothly and be less boring.



                  But writers do the same thing, envisioning the reader of the paper, or perhaps just the reviewers. If I say such a thing and the reviewer calls me on it, then I know I have to say more. But I trust that the reviewer can validly stand in place of the audience and will be able to easily fill such gaps.



                  When such a phrase is used (we can conclude X) it implies that X flows from the previous statement(s) and isn't just an unsupported statement inserted into the flow. It indicates a hopefully simple, but unstated, argument.



                  The alternative of including the complete argument is longer, more pedantic, more boring papers.



                  The other alternative of starting every sentence with "Therefore,..." is also stilted and, eventually, boring. Some variation of phrasing makes the reading more pleasant. Writers, in general, like to have a repertoire of, more or less, equivalent phrases to help the flow.



                  On the other hand, it has happened that the statement is used incorrectly when it takes a lot of argument and deep insight to go from point A to point B. Sometimes the author just doesn't notice the width of the gap. But, for the most part, it seems to work out.






                  share|improve this answer















                  Mathematical papers, especially, and many others, are not written for the complete novice with no understanding of the field. One tries to tailor the explanation of a new concept to the general level of understanding of the audience and tries to avoid being overly pedantic giving every detail and argument back to Euclid.



                  College lecturers often use such phrases, knowing the level that their students should have attained and, thus, permitting the lecture to flow more smoothly and be less boring.



                  But writers do the same thing, envisioning the reader of the paper, or perhaps just the reviewers. If I say such a thing and the reviewer calls me on it, then I know I have to say more. But I trust that the reviewer can validly stand in place of the audience and will be able to easily fill such gaps.



                  When such a phrase is used (we can conclude X) it implies that X flows from the previous statement(s) and isn't just an unsupported statement inserted into the flow. It indicates a hopefully simple, but unstated, argument.



                  The alternative of including the complete argument is longer, more pedantic, more boring papers.



                  The other alternative of starting every sentence with "Therefore,..." is also stilted and, eventually, boring. Some variation of phrasing makes the reading more pleasant. Writers, in general, like to have a repertoire of, more or less, equivalent phrases to help the flow.



                  On the other hand, it has happened that the statement is used incorrectly when it takes a lot of argument and deep insight to go from point A to point B. Sometimes the author just doesn't notice the width of the gap. But, for the most part, it seems to work out.







                  share|improve this answer














                  share|improve this answer



                  share|improve this answer








                  edited May 12 at 20:24

























                  answered May 12 at 12:37









                  BuffyBuffy

                  65.6k18201306




                  65.6k18201306








                  • 76





                    One wants to indicate that X flows from the previous and isn't just an unsupported statement inserted into the flow. The alternative (longer) is to include the complete argument.

                    – Buffy
                    May 12 at 12:47








                  • 41





                    +1 for and isn't just an unsupported statement inserted into the flow. One of the things I used to find so frustrating when reading physics texts (and worse yet, engineering, chemistry, etc.; in the case of the many social science texts I had to read for required "elective courses", it was actually not much of a concern, since pretty much everything in those texts was clearly non-deductive) was in trying to determine which author assertions were statements whose validity I was supposed to be able to determine based on prior material in the text (continued)

                    – Dave L Renfro
                    May 12 at 13:22








                  • 37





                    and which author assertions were simply assertions of the FYI variety. Regarding your answer, besides alerting the reader to the fact that an assertion is intended to be deducible by the reader, saying "it is easy to see that" (when used correctly) assists the reader in what to consider and how deep to go. For an analogy, I used to play chess a lot when I was young (mostly before high school), and it was MUCH easier to win over a weaker opponent if I KNEW the opponent was weak, because then I could go for higher risk moves that lead to quick capture of pieces.

                    – Dave L Renfro
                    May 12 at 13:31








                  • 19





                    Yes, +1 for distinguishing it from an unsupported - or even apparently unsupported - statement. In a math paper, if I come across what appears to be an unsupported statement in the flow of a proof, a common explanation is that it's something that was shown earlier in the paper that I've forgotten about. "It is easy to see" stops me going back and looking for it.

                    – Especially Lime
                    May 12 at 15:01








                  • 13





                    @ErelSegal-Halevi "It is easy to see X" is code for "I am skipping some steps, but X follows." I very much appreciate if authors let me know when they skip a few steps so that I won't try to find them in the text.

                    – Jair Taylor
                    May 12 at 20:34














                  • 76





                    One wants to indicate that X flows from the previous and isn't just an unsupported statement inserted into the flow. The alternative (longer) is to include the complete argument.

                    – Buffy
                    May 12 at 12:47








                  • 41





                    +1 for and isn't just an unsupported statement inserted into the flow. One of the things I used to find so frustrating when reading physics texts (and worse yet, engineering, chemistry, etc.; in the case of the many social science texts I had to read for required "elective courses", it was actually not much of a concern, since pretty much everything in those texts was clearly non-deductive) was in trying to determine which author assertions were statements whose validity I was supposed to be able to determine based on prior material in the text (continued)

                    – Dave L Renfro
                    May 12 at 13:22








                  • 37





                    and which author assertions were simply assertions of the FYI variety. Regarding your answer, besides alerting the reader to the fact that an assertion is intended to be deducible by the reader, saying "it is easy to see that" (when used correctly) assists the reader in what to consider and how deep to go. For an analogy, I used to play chess a lot when I was young (mostly before high school), and it was MUCH easier to win over a weaker opponent if I KNEW the opponent was weak, because then I could go for higher risk moves that lead to quick capture of pieces.

                    – Dave L Renfro
                    May 12 at 13:31








                  • 19





                    Yes, +1 for distinguishing it from an unsupported - or even apparently unsupported - statement. In a math paper, if I come across what appears to be an unsupported statement in the flow of a proof, a common explanation is that it's something that was shown earlier in the paper that I've forgotten about. "It is easy to see" stops me going back and looking for it.

                    – Especially Lime
                    May 12 at 15:01








                  • 13





                    @ErelSegal-Halevi "It is easy to see X" is code for "I am skipping some steps, but X follows." I very much appreciate if authors let me know when they skip a few steps so that I won't try to find them in the text.

                    – Jair Taylor
                    May 12 at 20:34








                  76




                  76





                  One wants to indicate that X flows from the previous and isn't just an unsupported statement inserted into the flow. The alternative (longer) is to include the complete argument.

                  – Buffy
                  May 12 at 12:47







                  One wants to indicate that X flows from the previous and isn't just an unsupported statement inserted into the flow. The alternative (longer) is to include the complete argument.

                  – Buffy
                  May 12 at 12:47






                  41




                  41





                  +1 for and isn't just an unsupported statement inserted into the flow. One of the things I used to find so frustrating when reading physics texts (and worse yet, engineering, chemistry, etc.; in the case of the many social science texts I had to read for required "elective courses", it was actually not much of a concern, since pretty much everything in those texts was clearly non-deductive) was in trying to determine which author assertions were statements whose validity I was supposed to be able to determine based on prior material in the text (continued)

                  – Dave L Renfro
                  May 12 at 13:22







                  +1 for and isn't just an unsupported statement inserted into the flow. One of the things I used to find so frustrating when reading physics texts (and worse yet, engineering, chemistry, etc.; in the case of the many social science texts I had to read for required "elective courses", it was actually not much of a concern, since pretty much everything in those texts was clearly non-deductive) was in trying to determine which author assertions were statements whose validity I was supposed to be able to determine based on prior material in the text (continued)

                  – Dave L Renfro
                  May 12 at 13:22






                  37




                  37





                  and which author assertions were simply assertions of the FYI variety. Regarding your answer, besides alerting the reader to the fact that an assertion is intended to be deducible by the reader, saying "it is easy to see that" (when used correctly) assists the reader in what to consider and how deep to go. For an analogy, I used to play chess a lot when I was young (mostly before high school), and it was MUCH easier to win over a weaker opponent if I KNEW the opponent was weak, because then I could go for higher risk moves that lead to quick capture of pieces.

                  – Dave L Renfro
                  May 12 at 13:31







                  and which author assertions were simply assertions of the FYI variety. Regarding your answer, besides alerting the reader to the fact that an assertion is intended to be deducible by the reader, saying "it is easy to see that" (when used correctly) assists the reader in what to consider and how deep to go. For an analogy, I used to play chess a lot when I was young (mostly before high school), and it was MUCH easier to win over a weaker opponent if I KNEW the opponent was weak, because then I could go for higher risk moves that lead to quick capture of pieces.

                  – Dave L Renfro
                  May 12 at 13:31






                  19




                  19





                  Yes, +1 for distinguishing it from an unsupported - or even apparently unsupported - statement. In a math paper, if I come across what appears to be an unsupported statement in the flow of a proof, a common explanation is that it's something that was shown earlier in the paper that I've forgotten about. "It is easy to see" stops me going back and looking for it.

                  – Especially Lime
                  May 12 at 15:01







                  Yes, +1 for distinguishing it from an unsupported - or even apparently unsupported - statement. In a math paper, if I come across what appears to be an unsupported statement in the flow of a proof, a common explanation is that it's something that was shown earlier in the paper that I've forgotten about. "It is easy to see" stops me going back and looking for it.

                  – Especially Lime
                  May 12 at 15:01






                  13




                  13





                  @ErelSegal-Halevi "It is easy to see X" is code for "I am skipping some steps, but X follows." I very much appreciate if authors let me know when they skip a few steps so that I won't try to find them in the text.

                  – Jair Taylor
                  May 12 at 20:34





                  @ErelSegal-Halevi "It is easy to see X" is code for "I am skipping some steps, but X follows." I very much appreciate if authors let me know when they skip a few steps so that I won't try to find them in the text.

                  – Jair Taylor
                  May 12 at 20:34











                  63














                  Several times during seminars I heard the following exchange, which sounds like a joke but actually isn't:



                  Audience member: Why is X true?



                  Speaker: Oh, it's obvious.



                  Audience member: OK, thanks! [sits down satisfied]





                  Especially in mathematics, it is often helpful to let the Reader know what level of difficulty and/or complexity each step is. The approach you (or at least I) would take differs significantly depending on this.



                  If a step is supposed to be easy and it's not quite my field, I'd take note that this sort of transition is standard in these parts and move on. Conversely, if the step is supposed to be easy and it is my field and it doesn't seem easy at all, then it's a strong indication that either the author is wrong (rare) or that my understanding is insufficient and I should think about this transition until it seems easy (not just until I find any way to justify it).



                  If a step is supposed to be moderate in difficulty, I might or might not work out the details myslef as an exercise, but I will also be aware that some amount of work goes into making it possible - helpful when evaluating if an argument is plausible and also in identifying where the content of the paper really is (especially in papers that are heavy on definitions, it's often a nontrivial task to figure out which transitions are easy but unfamiliar and which are responsible for the actual progress).



                  Finally, if a step is said to be difficult then there obviously should be a reference or a proof. If it's a proof, then more likely than not this is the point where the progress is being made in the paper, so if I'm reading the paper this is the part I would study in the most detail. If it's a reference, I would make a mental note that it's a potentially strong tool to keep in my arsenal - also, I would know better than to attempt to reproduce the result myself.



                  Note that it's not always all that easy to determine which is which without the Author explicitly making a judgement. The short phrase "By Thm. C in [42] we have X" could expand into either of "It is easy to see that X (see Thm C in [42] for details)" and "Because of the deep theorem of Smith (Thm C in [42]) we have X".





                  Having said all of the above, I want to add that personally, I dislike the phrase "It is easy to see". I understand the sentiment, but if the paper is read by anyone other than the experts in the field (maybe undergrads or experts in another field) then chances are it's not going to be easy to see to all the Readers, and the Readers who don't find it easy might feel bad about it in one way or another. If I'm already taking the time to explain myself for not explaining a transition I try to give some more details: either a super-short sketch of a proof, or a phrase like "It follows by an application of standard techniques that ..." or "A simple but mundane computation shows that...", etc.






                  share|improve this answer



















                  • 3





                    +1 for "Especially in mathematics, it is often helpful to let the Reader know what level of difficulty and/or complexity each step is." Sometimes it's difficult to know if the leap from one step to another is difficult or not. By saying "It's easy to see" you're letting the reader know that probably the first/simplest line of reasoning they attempt to bridge the gap is the correct one, and you therefore help the reader not waste too much mental energy in trying to determine the soundness of their ad-hoc reasoning, since it is most likely correct.

                    – ChocolateAndCheese
                    May 14 at 0:33






                  • 1





                    Just to clarify, the "OK, thanks" in the exchange does not mean "I get it", but more than "I trust you that you are right". When the audience does not get a thing, it's also natural to wonder if the speaker is really sure about that. Therefore, the beginning question "Why is X true?" really means "Are you sure that X is true?"

                    – Ooker
                    May 14 at 8:36






                  • 1





                    @Ooker - It's not so much about doubting the speaker (after all, they are the expert on the subject and have presumably spent some time preparing the talk and making sure that what they say is correct). It's more about the desire to understand why X is true. What the speaker is effectively saying is "I assure you that this transition is correct and that if you had all the relevant ingredients at hand and maybe spent 5 minutes with a piece of paper, you would easily reproduce it; there's nothing deep or particularly interesting happening here.".

                    – Jakub Konieczny
                    May 14 at 10:23






                  • 2





                    @Ooker It can even mean "I get it" (if preceeded by 5sec pausing). In informal discussions with colleagues I certainly had it happen that upon being told that something was obvious I could immediately see that it was, indeed, obvious, which had not been obvious to me before I asked.

                    – Arno
                    May 14 at 10:38






                  • 3





                    @Arno - That's a very fair point. As a slight variation on the same theme, there are often situations there there is a very obvious way to try and prove something, with the caveat that at each step something could potentially go wrong and become difficult. In a case like that, "It's obvious" might mean: The obvious argument works, nothing breaks down along the way.

                    – Jakub Konieczny
                    May 14 at 10:45
















                  63














                  Several times during seminars I heard the following exchange, which sounds like a joke but actually isn't:



                  Audience member: Why is X true?



                  Speaker: Oh, it's obvious.



                  Audience member: OK, thanks! [sits down satisfied]





                  Especially in mathematics, it is often helpful to let the Reader know what level of difficulty and/or complexity each step is. The approach you (or at least I) would take differs significantly depending on this.



                  If a step is supposed to be easy and it's not quite my field, I'd take note that this sort of transition is standard in these parts and move on. Conversely, if the step is supposed to be easy and it is my field and it doesn't seem easy at all, then it's a strong indication that either the author is wrong (rare) or that my understanding is insufficient and I should think about this transition until it seems easy (not just until I find any way to justify it).



                  If a step is supposed to be moderate in difficulty, I might or might not work out the details myslef as an exercise, but I will also be aware that some amount of work goes into making it possible - helpful when evaluating if an argument is plausible and also in identifying where the content of the paper really is (especially in papers that are heavy on definitions, it's often a nontrivial task to figure out which transitions are easy but unfamiliar and which are responsible for the actual progress).



                  Finally, if a step is said to be difficult then there obviously should be a reference or a proof. If it's a proof, then more likely than not this is the point where the progress is being made in the paper, so if I'm reading the paper this is the part I would study in the most detail. If it's a reference, I would make a mental note that it's a potentially strong tool to keep in my arsenal - also, I would know better than to attempt to reproduce the result myself.



                  Note that it's not always all that easy to determine which is which without the Author explicitly making a judgement. The short phrase "By Thm. C in [42] we have X" could expand into either of "It is easy to see that X (see Thm C in [42] for details)" and "Because of the deep theorem of Smith (Thm C in [42]) we have X".





                  Having said all of the above, I want to add that personally, I dislike the phrase "It is easy to see". I understand the sentiment, but if the paper is read by anyone other than the experts in the field (maybe undergrads or experts in another field) then chances are it's not going to be easy to see to all the Readers, and the Readers who don't find it easy might feel bad about it in one way or another. If I'm already taking the time to explain myself for not explaining a transition I try to give some more details: either a super-short sketch of a proof, or a phrase like "It follows by an application of standard techniques that ..." or "A simple but mundane computation shows that...", etc.






                  share|improve this answer



















                  • 3





                    +1 for "Especially in mathematics, it is often helpful to let the Reader know what level of difficulty and/or complexity each step is." Sometimes it's difficult to know if the leap from one step to another is difficult or not. By saying "It's easy to see" you're letting the reader know that probably the first/simplest line of reasoning they attempt to bridge the gap is the correct one, and you therefore help the reader not waste too much mental energy in trying to determine the soundness of their ad-hoc reasoning, since it is most likely correct.

                    – ChocolateAndCheese
                    May 14 at 0:33






                  • 1





                    Just to clarify, the "OK, thanks" in the exchange does not mean "I get it", but more than "I trust you that you are right". When the audience does not get a thing, it's also natural to wonder if the speaker is really sure about that. Therefore, the beginning question "Why is X true?" really means "Are you sure that X is true?"

                    – Ooker
                    May 14 at 8:36






                  • 1





                    @Ooker - It's not so much about doubting the speaker (after all, they are the expert on the subject and have presumably spent some time preparing the talk and making sure that what they say is correct). It's more about the desire to understand why X is true. What the speaker is effectively saying is "I assure you that this transition is correct and that if you had all the relevant ingredients at hand and maybe spent 5 minutes with a piece of paper, you would easily reproduce it; there's nothing deep or particularly interesting happening here.".

                    – Jakub Konieczny
                    May 14 at 10:23






                  • 2





                    @Ooker It can even mean "I get it" (if preceeded by 5sec pausing). In informal discussions with colleagues I certainly had it happen that upon being told that something was obvious I could immediately see that it was, indeed, obvious, which had not been obvious to me before I asked.

                    – Arno
                    May 14 at 10:38






                  • 3





                    @Arno - That's a very fair point. As a slight variation on the same theme, there are often situations there there is a very obvious way to try and prove something, with the caveat that at each step something could potentially go wrong and become difficult. In a case like that, "It's obvious" might mean: The obvious argument works, nothing breaks down along the way.

                    – Jakub Konieczny
                    May 14 at 10:45














                  63












                  63








                  63







                  Several times during seminars I heard the following exchange, which sounds like a joke but actually isn't:



                  Audience member: Why is X true?



                  Speaker: Oh, it's obvious.



                  Audience member: OK, thanks! [sits down satisfied]





                  Especially in mathematics, it is often helpful to let the Reader know what level of difficulty and/or complexity each step is. The approach you (or at least I) would take differs significantly depending on this.



                  If a step is supposed to be easy and it's not quite my field, I'd take note that this sort of transition is standard in these parts and move on. Conversely, if the step is supposed to be easy and it is my field and it doesn't seem easy at all, then it's a strong indication that either the author is wrong (rare) or that my understanding is insufficient and I should think about this transition until it seems easy (not just until I find any way to justify it).



                  If a step is supposed to be moderate in difficulty, I might or might not work out the details myslef as an exercise, but I will also be aware that some amount of work goes into making it possible - helpful when evaluating if an argument is plausible and also in identifying where the content of the paper really is (especially in papers that are heavy on definitions, it's often a nontrivial task to figure out which transitions are easy but unfamiliar and which are responsible for the actual progress).



                  Finally, if a step is said to be difficult then there obviously should be a reference or a proof. If it's a proof, then more likely than not this is the point where the progress is being made in the paper, so if I'm reading the paper this is the part I would study in the most detail. If it's a reference, I would make a mental note that it's a potentially strong tool to keep in my arsenal - also, I would know better than to attempt to reproduce the result myself.



                  Note that it's not always all that easy to determine which is which without the Author explicitly making a judgement. The short phrase "By Thm. C in [42] we have X" could expand into either of "It is easy to see that X (see Thm C in [42] for details)" and "Because of the deep theorem of Smith (Thm C in [42]) we have X".





                  Having said all of the above, I want to add that personally, I dislike the phrase "It is easy to see". I understand the sentiment, but if the paper is read by anyone other than the experts in the field (maybe undergrads or experts in another field) then chances are it's not going to be easy to see to all the Readers, and the Readers who don't find it easy might feel bad about it in one way or another. If I'm already taking the time to explain myself for not explaining a transition I try to give some more details: either a super-short sketch of a proof, or a phrase like "It follows by an application of standard techniques that ..." or "A simple but mundane computation shows that...", etc.






                  share|improve this answer













                  Several times during seminars I heard the following exchange, which sounds like a joke but actually isn't:



                  Audience member: Why is X true?



                  Speaker: Oh, it's obvious.



                  Audience member: OK, thanks! [sits down satisfied]





                  Especially in mathematics, it is often helpful to let the Reader know what level of difficulty and/or complexity each step is. The approach you (or at least I) would take differs significantly depending on this.



                  If a step is supposed to be easy and it's not quite my field, I'd take note that this sort of transition is standard in these parts and move on. Conversely, if the step is supposed to be easy and it is my field and it doesn't seem easy at all, then it's a strong indication that either the author is wrong (rare) or that my understanding is insufficient and I should think about this transition until it seems easy (not just until I find any way to justify it).



                  If a step is supposed to be moderate in difficulty, I might or might not work out the details myslef as an exercise, but I will also be aware that some amount of work goes into making it possible - helpful when evaluating if an argument is plausible and also in identifying where the content of the paper really is (especially in papers that are heavy on definitions, it's often a nontrivial task to figure out which transitions are easy but unfamiliar and which are responsible for the actual progress).



                  Finally, if a step is said to be difficult then there obviously should be a reference or a proof. If it's a proof, then more likely than not this is the point where the progress is being made in the paper, so if I'm reading the paper this is the part I would study in the most detail. If it's a reference, I would make a mental note that it's a potentially strong tool to keep in my arsenal - also, I would know better than to attempt to reproduce the result myself.



                  Note that it's not always all that easy to determine which is which without the Author explicitly making a judgement. The short phrase "By Thm. C in [42] we have X" could expand into either of "It is easy to see that X (see Thm C in [42] for details)" and "Because of the deep theorem of Smith (Thm C in [42]) we have X".





                  Having said all of the above, I want to add that personally, I dislike the phrase "It is easy to see". I understand the sentiment, but if the paper is read by anyone other than the experts in the field (maybe undergrads or experts in another field) then chances are it's not going to be easy to see to all the Readers, and the Readers who don't find it easy might feel bad about it in one way or another. If I'm already taking the time to explain myself for not explaining a transition I try to give some more details: either a super-short sketch of a proof, or a phrase like "It follows by an application of standard techniques that ..." or "A simple but mundane computation shows that...", etc.







                  share|improve this answer












                  share|improve this answer



                  share|improve this answer










                  answered May 12 at 18:32









                  Jakub KoniecznyJakub Konieczny

                  3,16712038




                  3,16712038








                  • 3





                    +1 for "Especially in mathematics, it is often helpful to let the Reader know what level of difficulty and/or complexity each step is." Sometimes it's difficult to know if the leap from one step to another is difficult or not. By saying "It's easy to see" you're letting the reader know that probably the first/simplest line of reasoning they attempt to bridge the gap is the correct one, and you therefore help the reader not waste too much mental energy in trying to determine the soundness of their ad-hoc reasoning, since it is most likely correct.

                    – ChocolateAndCheese
                    May 14 at 0:33






                  • 1





                    Just to clarify, the "OK, thanks" in the exchange does not mean "I get it", but more than "I trust you that you are right". When the audience does not get a thing, it's also natural to wonder if the speaker is really sure about that. Therefore, the beginning question "Why is X true?" really means "Are you sure that X is true?"

                    – Ooker
                    May 14 at 8:36






                  • 1





                    @Ooker - It's not so much about doubting the speaker (after all, they are the expert on the subject and have presumably spent some time preparing the talk and making sure that what they say is correct). It's more about the desire to understand why X is true. What the speaker is effectively saying is "I assure you that this transition is correct and that if you had all the relevant ingredients at hand and maybe spent 5 minutes with a piece of paper, you would easily reproduce it; there's nothing deep or particularly interesting happening here.".

                    – Jakub Konieczny
                    May 14 at 10:23






                  • 2





                    @Ooker It can even mean "I get it" (if preceeded by 5sec pausing). In informal discussions with colleagues I certainly had it happen that upon being told that something was obvious I could immediately see that it was, indeed, obvious, which had not been obvious to me before I asked.

                    – Arno
                    May 14 at 10:38






                  • 3





                    @Arno - That's a very fair point. As a slight variation on the same theme, there are often situations there there is a very obvious way to try and prove something, with the caveat that at each step something could potentially go wrong and become difficult. In a case like that, "It's obvious" might mean: The obvious argument works, nothing breaks down along the way.

                    – Jakub Konieczny
                    May 14 at 10:45














                  • 3





                    +1 for "Especially in mathematics, it is often helpful to let the Reader know what level of difficulty and/or complexity each step is." Sometimes it's difficult to know if the leap from one step to another is difficult or not. By saying "It's easy to see" you're letting the reader know that probably the first/simplest line of reasoning they attempt to bridge the gap is the correct one, and you therefore help the reader not waste too much mental energy in trying to determine the soundness of their ad-hoc reasoning, since it is most likely correct.

                    – ChocolateAndCheese
                    May 14 at 0:33






                  • 1





                    Just to clarify, the "OK, thanks" in the exchange does not mean "I get it", but more than "I trust you that you are right". When the audience does not get a thing, it's also natural to wonder if the speaker is really sure about that. Therefore, the beginning question "Why is X true?" really means "Are you sure that X is true?"

                    – Ooker
                    May 14 at 8:36






                  • 1





                    @Ooker - It's not so much about doubting the speaker (after all, they are the expert on the subject and have presumably spent some time preparing the talk and making sure that what they say is correct). It's more about the desire to understand why X is true. What the speaker is effectively saying is "I assure you that this transition is correct and that if you had all the relevant ingredients at hand and maybe spent 5 minutes with a piece of paper, you would easily reproduce it; there's nothing deep or particularly interesting happening here.".

                    – Jakub Konieczny
                    May 14 at 10:23






                  • 2





                    @Ooker It can even mean "I get it" (if preceeded by 5sec pausing). In informal discussions with colleagues I certainly had it happen that upon being told that something was obvious I could immediately see that it was, indeed, obvious, which had not been obvious to me before I asked.

                    – Arno
                    May 14 at 10:38






                  • 3





                    @Arno - That's a very fair point. As a slight variation on the same theme, there are often situations there there is a very obvious way to try and prove something, with the caveat that at each step something could potentially go wrong and become difficult. In a case like that, "It's obvious" might mean: The obvious argument works, nothing breaks down along the way.

                    – Jakub Konieczny
                    May 14 at 10:45








                  3




                  3





                  +1 for "Especially in mathematics, it is often helpful to let the Reader know what level of difficulty and/or complexity each step is." Sometimes it's difficult to know if the leap from one step to another is difficult or not. By saying "It's easy to see" you're letting the reader know that probably the first/simplest line of reasoning they attempt to bridge the gap is the correct one, and you therefore help the reader not waste too much mental energy in trying to determine the soundness of their ad-hoc reasoning, since it is most likely correct.

                  – ChocolateAndCheese
                  May 14 at 0:33





                  +1 for "Especially in mathematics, it is often helpful to let the Reader know what level of difficulty and/or complexity each step is." Sometimes it's difficult to know if the leap from one step to another is difficult or not. By saying "It's easy to see" you're letting the reader know that probably the first/simplest line of reasoning they attempt to bridge the gap is the correct one, and you therefore help the reader not waste too much mental energy in trying to determine the soundness of their ad-hoc reasoning, since it is most likely correct.

                  – ChocolateAndCheese
                  May 14 at 0:33




                  1




                  1





                  Just to clarify, the "OK, thanks" in the exchange does not mean "I get it", but more than "I trust you that you are right". When the audience does not get a thing, it's also natural to wonder if the speaker is really sure about that. Therefore, the beginning question "Why is X true?" really means "Are you sure that X is true?"

                  – Ooker
                  May 14 at 8:36





                  Just to clarify, the "OK, thanks" in the exchange does not mean "I get it", but more than "I trust you that you are right". When the audience does not get a thing, it's also natural to wonder if the speaker is really sure about that. Therefore, the beginning question "Why is X true?" really means "Are you sure that X is true?"

                  – Ooker
                  May 14 at 8:36




                  1




                  1





                  @Ooker - It's not so much about doubting the speaker (after all, they are the expert on the subject and have presumably spent some time preparing the talk and making sure that what they say is correct). It's more about the desire to understand why X is true. What the speaker is effectively saying is "I assure you that this transition is correct and that if you had all the relevant ingredients at hand and maybe spent 5 minutes with a piece of paper, you would easily reproduce it; there's nothing deep or particularly interesting happening here.".

                  – Jakub Konieczny
                  May 14 at 10:23





                  @Ooker - It's not so much about doubting the speaker (after all, they are the expert on the subject and have presumably spent some time preparing the talk and making sure that what they say is correct). It's more about the desire to understand why X is true. What the speaker is effectively saying is "I assure you that this transition is correct and that if you had all the relevant ingredients at hand and maybe spent 5 minutes with a piece of paper, you would easily reproduce it; there's nothing deep or particularly interesting happening here.".

                  – Jakub Konieczny
                  May 14 at 10:23




                  2




                  2





                  @Ooker It can even mean "I get it" (if preceeded by 5sec pausing). In informal discussions with colleagues I certainly had it happen that upon being told that something was obvious I could immediately see that it was, indeed, obvious, which had not been obvious to me before I asked.

                  – Arno
                  May 14 at 10:38





                  @Ooker It can even mean "I get it" (if preceeded by 5sec pausing). In informal discussions with colleagues I certainly had it happen that upon being told that something was obvious I could immediately see that it was, indeed, obvious, which had not been obvious to me before I asked.

                  – Arno
                  May 14 at 10:38




                  3




                  3





                  @Arno - That's a very fair point. As a slight variation on the same theme, there are often situations there there is a very obvious way to try and prove something, with the caveat that at each step something could potentially go wrong and become difficult. In a case like that, "It's obvious" might mean: The obvious argument works, nothing breaks down along the way.

                  – Jakub Konieczny
                  May 14 at 10:45





                  @Arno - That's a very fair point. As a slight variation on the same theme, there are often situations there there is a very obvious way to try and prove something, with the caveat that at each step something could potentially go wrong and become difficult. In a case like that, "It's obvious" might mean: The obvious argument works, nothing breaks down along the way.

                  – Jakub Konieczny
                  May 14 at 10:45











                  15














                  There are no good reasons for phrases like that one. If something is "easy to see" or is "obvious", then there is no point in emphasizing such truism.



                  However, if no additional knowledge in needed and the following steps are quite straightforward, then one can use the advice form the Mathematical writing article in The Princeton companion to applied mathematics:




                  The question of how much detail to give is related to the question of
                  how formal to be, but it is not the same question. It is true that
                  there is a tendency in informal mathematical writing to leave out
                  details, but with even the most formal writing a decision has to be
                  made about how much detail to give; it is just that in formal writing
                  one probably wants to signal more carefully when details have been
                  left out. This can be done in various ways. One can use expressions
                  such as “It is an easy exercise to check that...,” or “The second case
                  is similar,” which basically say to the reader, “I have decided not to
                  spell out this part of the argument.” One can also give small hints,
                  such as “By compactness,” or “An obvious inductive argument now
                  shows that...,” or “Interchanging the order of summation and
                  simplifying, we obtain....”



                  If you do decide to leave out detail, it
                  is a good idea to signal to the reader how difficult it would be to
                  put that detail in. A mistake that some writers make is to give
                  references to other papers for arguments that can easily be worked
                  out by the reader, without saying that the particular result that is
                  needed is easy. This is straight- forwardly misleading; it suggests
                  that the best thing to do is to go and look up the other paper when in
                  fact the best thing to do is to work out the argument for oneself.




                  Also from How to read and understand a paper in the same book:




                  In mathematical writing certain standard phrases are used that have
                  particular meanings. “It follows that” or “it is easy to see that”
                  mean that the next statement can be proved without using any new ideas

                  and that giving the details would clutter the text. The detail may,
                  however, be tedious. The shorter “hence,” “therefore,” or “so” imply a
                  more straightforward conclusion. “It can be shown that” again implies
                  that details are not felt to be worth including but is noncommittal
                  about the difficulty of the proof.







                  share|improve this answer





















                  • 1





                    From a strictly grammatical standpoint, yes, you're correct. But humans often need nudges to connect ideas mentally. It lowers the cognition required to process the more important details at hand.

                    – Carl V. Lewis
                    May 12 at 20:59






                  • 3





                    Such phrases are not pointless, because they provide metadata. Language gives us not only the ability to transfer information to others, and also information about that information, and even information about the information about that information. It's very common to use language to convey meaning on multiple levels at once.

                    – barbecue
                    May 12 at 21:03






                  • 3





                    @CarlV.Lewis This is exactly why it is better give more useful breadcrumbs to connect ideas, e.g.: "From the application of Theorem 4.1 to system 2.4, it is easy to see..."

                    – homocomputeris
                    May 12 at 22:46






                  • 10





                    "There are no good reasons..." ... proceeding several good reasons. ;)

                    – AnoE
                    May 13 at 16:07






                  • 3





                    If something is "easy to see" but I don't see it, that is a very useful cue for me to go back and check if I missed something simple, rather than trying more complicated methods of deriving the "obvious" result. A simple line of reasoning is easier to deduce if you can exclude the more complicated ones. Even if X is obvious to most people, it will be obvious to even more people if you label it as an "obvious result".

                    – Nuclear Wang
                    May 13 at 19:05
















                  15














                  There are no good reasons for phrases like that one. If something is "easy to see" or is "obvious", then there is no point in emphasizing such truism.



                  However, if no additional knowledge in needed and the following steps are quite straightforward, then one can use the advice form the Mathematical writing article in The Princeton companion to applied mathematics:




                  The question of how much detail to give is related to the question of
                  how formal to be, but it is not the same question. It is true that
                  there is a tendency in informal mathematical writing to leave out
                  details, but with even the most formal writing a decision has to be
                  made about how much detail to give; it is just that in formal writing
                  one probably wants to signal more carefully when details have been
                  left out. This can be done in various ways. One can use expressions
                  such as “It is an easy exercise to check that...,” or “The second case
                  is similar,” which basically say to the reader, “I have decided not to
                  spell out this part of the argument.” One can also give small hints,
                  such as “By compactness,” or “An obvious inductive argument now
                  shows that...,” or “Interchanging the order of summation and
                  simplifying, we obtain....”



                  If you do decide to leave out detail, it
                  is a good idea to signal to the reader how difficult it would be to
                  put that detail in. A mistake that some writers make is to give
                  references to other papers for arguments that can easily be worked
                  out by the reader, without saying that the particular result that is
                  needed is easy. This is straight- forwardly misleading; it suggests
                  that the best thing to do is to go and look up the other paper when in
                  fact the best thing to do is to work out the argument for oneself.




                  Also from How to read and understand a paper in the same book:




                  In mathematical writing certain standard phrases are used that have
                  particular meanings. “It follows that” or “it is easy to see that”
                  mean that the next statement can be proved without using any new ideas

                  and that giving the details would clutter the text. The detail may,
                  however, be tedious. The shorter “hence,” “therefore,” or “so” imply a
                  more straightforward conclusion. “It can be shown that” again implies
                  that details are not felt to be worth including but is noncommittal
                  about the difficulty of the proof.







                  share|improve this answer





















                  • 1





                    From a strictly grammatical standpoint, yes, you're correct. But humans often need nudges to connect ideas mentally. It lowers the cognition required to process the more important details at hand.

                    – Carl V. Lewis
                    May 12 at 20:59






                  • 3





                    Such phrases are not pointless, because they provide metadata. Language gives us not only the ability to transfer information to others, and also information about that information, and even information about the information about that information. It's very common to use language to convey meaning on multiple levels at once.

                    – barbecue
                    May 12 at 21:03






                  • 3





                    @CarlV.Lewis This is exactly why it is better give more useful breadcrumbs to connect ideas, e.g.: "From the application of Theorem 4.1 to system 2.4, it is easy to see..."

                    – homocomputeris
                    May 12 at 22:46






                  • 10





                    "There are no good reasons..." ... proceeding several good reasons. ;)

                    – AnoE
                    May 13 at 16:07






                  • 3





                    If something is "easy to see" but I don't see it, that is a very useful cue for me to go back and check if I missed something simple, rather than trying more complicated methods of deriving the "obvious" result. A simple line of reasoning is easier to deduce if you can exclude the more complicated ones. Even if X is obvious to most people, it will be obvious to even more people if you label it as an "obvious result".

                    – Nuclear Wang
                    May 13 at 19:05














                  15












                  15








                  15







                  There are no good reasons for phrases like that one. If something is "easy to see" or is "obvious", then there is no point in emphasizing such truism.



                  However, if no additional knowledge in needed and the following steps are quite straightforward, then one can use the advice form the Mathematical writing article in The Princeton companion to applied mathematics:




                  The question of how much detail to give is related to the question of
                  how formal to be, but it is not the same question. It is true that
                  there is a tendency in informal mathematical writing to leave out
                  details, but with even the most formal writing a decision has to be
                  made about how much detail to give; it is just that in formal writing
                  one probably wants to signal more carefully when details have been
                  left out. This can be done in various ways. One can use expressions
                  such as “It is an easy exercise to check that...,” or “The second case
                  is similar,” which basically say to the reader, “I have decided not to
                  spell out this part of the argument.” One can also give small hints,
                  such as “By compactness,” or “An obvious inductive argument now
                  shows that...,” or “Interchanging the order of summation and
                  simplifying, we obtain....”



                  If you do decide to leave out detail, it
                  is a good idea to signal to the reader how difficult it would be to
                  put that detail in. A mistake that some writers make is to give
                  references to other papers for arguments that can easily be worked
                  out by the reader, without saying that the particular result that is
                  needed is easy. This is straight- forwardly misleading; it suggests
                  that the best thing to do is to go and look up the other paper when in
                  fact the best thing to do is to work out the argument for oneself.




                  Also from How to read and understand a paper in the same book:




                  In mathematical writing certain standard phrases are used that have
                  particular meanings. “It follows that” or “it is easy to see that”
                  mean that the next statement can be proved without using any new ideas

                  and that giving the details would clutter the text. The detail may,
                  however, be tedious. The shorter “hence,” “therefore,” or “so” imply a
                  more straightforward conclusion. “It can be shown that” again implies
                  that details are not felt to be worth including but is noncommittal
                  about the difficulty of the proof.







                  share|improve this answer















                  There are no good reasons for phrases like that one. If something is "easy to see" or is "obvious", then there is no point in emphasizing such truism.



                  However, if no additional knowledge in needed and the following steps are quite straightforward, then one can use the advice form the Mathematical writing article in The Princeton companion to applied mathematics:




                  The question of how much detail to give is related to the question of
                  how formal to be, but it is not the same question. It is true that
                  there is a tendency in informal mathematical writing to leave out
                  details, but with even the most formal writing a decision has to be
                  made about how much detail to give; it is just that in formal writing
                  one probably wants to signal more carefully when details have been
                  left out. This can be done in various ways. One can use expressions
                  such as “It is an easy exercise to check that...,” or “The second case
                  is similar,” which basically say to the reader, “I have decided not to
                  spell out this part of the argument.” One can also give small hints,
                  such as “By compactness,” or “An obvious inductive argument now
                  shows that...,” or “Interchanging the order of summation and
                  simplifying, we obtain....”



                  If you do decide to leave out detail, it
                  is a good idea to signal to the reader how difficult it would be to
                  put that detail in. A mistake that some writers make is to give
                  references to other papers for arguments that can easily be worked
                  out by the reader, without saying that the particular result that is
                  needed is easy. This is straight- forwardly misleading; it suggests
                  that the best thing to do is to go and look up the other paper when in
                  fact the best thing to do is to work out the argument for oneself.




                  Also from How to read and understand a paper in the same book:




                  In mathematical writing certain standard phrases are used that have
                  particular meanings. “It follows that” or “it is easy to see that”
                  mean that the next statement can be proved without using any new ideas

                  and that giving the details would clutter the text. The detail may,
                  however, be tedious. The shorter “hence,” “therefore,” or “so” imply a
                  more straightforward conclusion. “It can be shown that” again implies
                  that details are not felt to be worth including but is noncommittal
                  about the difficulty of the proof.








                  share|improve this answer














                  share|improve this answer



                  share|improve this answer








                  edited May 12 at 22:47









                  Randall Stewart

                  1493




                  1493










                  answered May 12 at 20:10









                  homocomputerishomocomputeris

                  408210




                  408210








                  • 1





                    From a strictly grammatical standpoint, yes, you're correct. But humans often need nudges to connect ideas mentally. It lowers the cognition required to process the more important details at hand.

                    – Carl V. Lewis
                    May 12 at 20:59






                  • 3





                    Such phrases are not pointless, because they provide metadata. Language gives us not only the ability to transfer information to others, and also information about that information, and even information about the information about that information. It's very common to use language to convey meaning on multiple levels at once.

                    – barbecue
                    May 12 at 21:03






                  • 3





                    @CarlV.Lewis This is exactly why it is better give more useful breadcrumbs to connect ideas, e.g.: "From the application of Theorem 4.1 to system 2.4, it is easy to see..."

                    – homocomputeris
                    May 12 at 22:46






                  • 10





                    "There are no good reasons..." ... proceeding several good reasons. ;)

                    – AnoE
                    May 13 at 16:07






                  • 3





                    If something is "easy to see" but I don't see it, that is a very useful cue for me to go back and check if I missed something simple, rather than trying more complicated methods of deriving the "obvious" result. A simple line of reasoning is easier to deduce if you can exclude the more complicated ones. Even if X is obvious to most people, it will be obvious to even more people if you label it as an "obvious result".

                    – Nuclear Wang
                    May 13 at 19:05














                  • 1





                    From a strictly grammatical standpoint, yes, you're correct. But humans often need nudges to connect ideas mentally. It lowers the cognition required to process the more important details at hand.

                    – Carl V. Lewis
                    May 12 at 20:59






                  • 3





                    Such phrases are not pointless, because they provide metadata. Language gives us not only the ability to transfer information to others, and also information about that information, and even information about the information about that information. It's very common to use language to convey meaning on multiple levels at once.

                    – barbecue
                    May 12 at 21:03






                  • 3





                    @CarlV.Lewis This is exactly why it is better give more useful breadcrumbs to connect ideas, e.g.: "From the application of Theorem 4.1 to system 2.4, it is easy to see..."

                    – homocomputeris
                    May 12 at 22:46






                  • 10





                    "There are no good reasons..." ... proceeding several good reasons. ;)

                    – AnoE
                    May 13 at 16:07






                  • 3





                    If something is "easy to see" but I don't see it, that is a very useful cue for me to go back and check if I missed something simple, rather than trying more complicated methods of deriving the "obvious" result. A simple line of reasoning is easier to deduce if you can exclude the more complicated ones. Even if X is obvious to most people, it will be obvious to even more people if you label it as an "obvious result".

                    – Nuclear Wang
                    May 13 at 19:05








                  1




                  1





                  From a strictly grammatical standpoint, yes, you're correct. But humans often need nudges to connect ideas mentally. It lowers the cognition required to process the more important details at hand.

                  – Carl V. Lewis
                  May 12 at 20:59





                  From a strictly grammatical standpoint, yes, you're correct. But humans often need nudges to connect ideas mentally. It lowers the cognition required to process the more important details at hand.

                  – Carl V. Lewis
                  May 12 at 20:59




                  3




                  3





                  Such phrases are not pointless, because they provide metadata. Language gives us not only the ability to transfer information to others, and also information about that information, and even information about the information about that information. It's very common to use language to convey meaning on multiple levels at once.

                  – barbecue
                  May 12 at 21:03





                  Such phrases are not pointless, because they provide metadata. Language gives us not only the ability to transfer information to others, and also information about that information, and even information about the information about that information. It's very common to use language to convey meaning on multiple levels at once.

                  – barbecue
                  May 12 at 21:03




                  3




                  3





                  @CarlV.Lewis This is exactly why it is better give more useful breadcrumbs to connect ideas, e.g.: "From the application of Theorem 4.1 to system 2.4, it is easy to see..."

                  – homocomputeris
                  May 12 at 22:46





                  @CarlV.Lewis This is exactly why it is better give more useful breadcrumbs to connect ideas, e.g.: "From the application of Theorem 4.1 to system 2.4, it is easy to see..."

                  – homocomputeris
                  May 12 at 22:46




                  10




                  10





                  "There are no good reasons..." ... proceeding several good reasons. ;)

                  – AnoE
                  May 13 at 16:07





                  "There are no good reasons..." ... proceeding several good reasons. ;)

                  – AnoE
                  May 13 at 16:07




                  3




                  3





                  If something is "easy to see" but I don't see it, that is a very useful cue for me to go back and check if I missed something simple, rather than trying more complicated methods of deriving the "obvious" result. A simple line of reasoning is easier to deduce if you can exclude the more complicated ones. Even if X is obvious to most people, it will be obvious to even more people if you label it as an "obvious result".

                  – Nuclear Wang
                  May 13 at 19:05





                  If something is "easy to see" but I don't see it, that is a very useful cue for me to go back and check if I missed something simple, rather than trying more complicated methods of deriving the "obvious" result. A simple line of reasoning is easier to deduce if you can exclude the more complicated ones. Even if X is obvious to most people, it will be obvious to even more people if you label it as an "obvious result".

                  – Nuclear Wang
                  May 13 at 19:05











                  13














                  One common way that I interpret "It is easy to see X" is as an ever so slight shortening of "it easily follows that X" (which I prefer). This itself is a shortened version of "The preceding assertion implies X", or perhaps a pluralized version such as "The preceding assertions (or definitions, or assertions and definitions, or...) imply X".



                  There is information contained here, namely a logical implication, that would be lost if one simply wrote "X".






                  share|improve this answer




























                    13














                    One common way that I interpret "It is easy to see X" is as an ever so slight shortening of "it easily follows that X" (which I prefer). This itself is a shortened version of "The preceding assertion implies X", or perhaps a pluralized version such as "The preceding assertions (or definitions, or assertions and definitions, or...) imply X".



                    There is information contained here, namely a logical implication, that would be lost if one simply wrote "X".






                    share|improve this answer


























                      13












                      13








                      13







                      One common way that I interpret "It is easy to see X" is as an ever so slight shortening of "it easily follows that X" (which I prefer). This itself is a shortened version of "The preceding assertion implies X", or perhaps a pluralized version such as "The preceding assertions (or definitions, or assertions and definitions, or...) imply X".



                      There is information contained here, namely a logical implication, that would be lost if one simply wrote "X".






                      share|improve this answer













                      One common way that I interpret "It is easy to see X" is as an ever so slight shortening of "it easily follows that X" (which I prefer). This itself is a shortened version of "The preceding assertion implies X", or perhaps a pluralized version such as "The preceding assertions (or definitions, or assertions and definitions, or...) imply X".



                      There is information contained here, namely a logical implication, that would be lost if one simply wrote "X".







                      share|improve this answer












                      share|improve this answer



                      share|improve this answer










                      answered May 12 at 17:01









                      Lee MosherLee Mosher

                      29915




                      29915























                          10














                          "It is easy to see" should be used sparingly. If one can say how it is easy to see why the statement is true, in a similar number of words, then this strategy is prefered. Alternative, but more specific expressions, than "it is easy to see" include




                          • After routine algebra, ...

                          • By theorem X, ...

                          • By definition of Y, ...


                          Note the phrase "It is easy to see that" is actually longer than all of the above.



                          Sometimes the reason why it is easy to see something cannot be explained in so few words. In which case, if it is truly easy to see for the target audience, using this phrase is useful to signal to the reader that if they don't follow, its probably some trivial thing that they've messed up in their head, rather than a need to think deeply about the statement. But even then, the phrase should only be used when it can't be made more specific.






                          share|improve this answer



















                          • 3





                            Your suggestion to include short specific suggestive methods of proof is something I began realizing many years ago had little effect on verbal length or content tangency while greatly helping some readers, and since then I've tried to do this as much as is reasonable, often adding such explanatory phrases on the second or third rewrite, as I don't always think about it the first time around. Other examples: Instead of "by an easy integration", say "letting u = ..." or "using integration by parts". Instead of "it is easy to show convergence", say "by limit comparison with a geometric series".

                            – Dave L Renfro
                            May 14 at 10:24











                          • @DaveLRenfro Agreed, but at least "by easy integration" is better than easy to see. It at least signals to the reader that, awe I need to compute this integral to get to the next statement, the more specific the better assuming not many words are required. Absolutely agree.

                            – WetlabStudent
                            May 15 at 3:50
















                          10














                          "It is easy to see" should be used sparingly. If one can say how it is easy to see why the statement is true, in a similar number of words, then this strategy is prefered. Alternative, but more specific expressions, than "it is easy to see" include




                          • After routine algebra, ...

                          • By theorem X, ...

                          • By definition of Y, ...


                          Note the phrase "It is easy to see that" is actually longer than all of the above.



                          Sometimes the reason why it is easy to see something cannot be explained in so few words. In which case, if it is truly easy to see for the target audience, using this phrase is useful to signal to the reader that if they don't follow, its probably some trivial thing that they've messed up in their head, rather than a need to think deeply about the statement. But even then, the phrase should only be used when it can't be made more specific.






                          share|improve this answer



















                          • 3





                            Your suggestion to include short specific suggestive methods of proof is something I began realizing many years ago had little effect on verbal length or content tangency while greatly helping some readers, and since then I've tried to do this as much as is reasonable, often adding such explanatory phrases on the second or third rewrite, as I don't always think about it the first time around. Other examples: Instead of "by an easy integration", say "letting u = ..." or "using integration by parts". Instead of "it is easy to show convergence", say "by limit comparison with a geometric series".

                            – Dave L Renfro
                            May 14 at 10:24











                          • @DaveLRenfro Agreed, but at least "by easy integration" is better than easy to see. It at least signals to the reader that, awe I need to compute this integral to get to the next statement, the more specific the better assuming not many words are required. Absolutely agree.

                            – WetlabStudent
                            May 15 at 3:50














                          10












                          10








                          10







                          "It is easy to see" should be used sparingly. If one can say how it is easy to see why the statement is true, in a similar number of words, then this strategy is prefered. Alternative, but more specific expressions, than "it is easy to see" include




                          • After routine algebra, ...

                          • By theorem X, ...

                          • By definition of Y, ...


                          Note the phrase "It is easy to see that" is actually longer than all of the above.



                          Sometimes the reason why it is easy to see something cannot be explained in so few words. In which case, if it is truly easy to see for the target audience, using this phrase is useful to signal to the reader that if they don't follow, its probably some trivial thing that they've messed up in their head, rather than a need to think deeply about the statement. But even then, the phrase should only be used when it can't be made more specific.






                          share|improve this answer













                          "It is easy to see" should be used sparingly. If one can say how it is easy to see why the statement is true, in a similar number of words, then this strategy is prefered. Alternative, but more specific expressions, than "it is easy to see" include




                          • After routine algebra, ...

                          • By theorem X, ...

                          • By definition of Y, ...


                          Note the phrase "It is easy to see that" is actually longer than all of the above.



                          Sometimes the reason why it is easy to see something cannot be explained in so few words. In which case, if it is truly easy to see for the target audience, using this phrase is useful to signal to the reader that if they don't follow, its probably some trivial thing that they've messed up in their head, rather than a need to think deeply about the statement. But even then, the phrase should only be used when it can't be made more specific.







                          share|improve this answer












                          share|improve this answer



                          share|improve this answer










                          answered May 13 at 1:43









                          WetlabStudentWetlabStudent

                          6,98332953




                          6,98332953








                          • 3





                            Your suggestion to include short specific suggestive methods of proof is something I began realizing many years ago had little effect on verbal length or content tangency while greatly helping some readers, and since then I've tried to do this as much as is reasonable, often adding such explanatory phrases on the second or third rewrite, as I don't always think about it the first time around. Other examples: Instead of "by an easy integration", say "letting u = ..." or "using integration by parts". Instead of "it is easy to show convergence", say "by limit comparison with a geometric series".

                            – Dave L Renfro
                            May 14 at 10:24











                          • @DaveLRenfro Agreed, but at least "by easy integration" is better than easy to see. It at least signals to the reader that, awe I need to compute this integral to get to the next statement, the more specific the better assuming not many words are required. Absolutely agree.

                            – WetlabStudent
                            May 15 at 3:50














                          • 3





                            Your suggestion to include short specific suggestive methods of proof is something I began realizing many years ago had little effect on verbal length or content tangency while greatly helping some readers, and since then I've tried to do this as much as is reasonable, often adding such explanatory phrases on the second or third rewrite, as I don't always think about it the first time around. Other examples: Instead of "by an easy integration", say "letting u = ..." or "using integration by parts". Instead of "it is easy to show convergence", say "by limit comparison with a geometric series".

                            – Dave L Renfro
                            May 14 at 10:24











                          • @DaveLRenfro Agreed, but at least "by easy integration" is better than easy to see. It at least signals to the reader that, awe I need to compute this integral to get to the next statement, the more specific the better assuming not many words are required. Absolutely agree.

                            – WetlabStudent
                            May 15 at 3:50








                          3




                          3





                          Your suggestion to include short specific suggestive methods of proof is something I began realizing many years ago had little effect on verbal length or content tangency while greatly helping some readers, and since then I've tried to do this as much as is reasonable, often adding such explanatory phrases on the second or third rewrite, as I don't always think about it the first time around. Other examples: Instead of "by an easy integration", say "letting u = ..." or "using integration by parts". Instead of "it is easy to show convergence", say "by limit comparison with a geometric series".

                          – Dave L Renfro
                          May 14 at 10:24





                          Your suggestion to include short specific suggestive methods of proof is something I began realizing many years ago had little effect on verbal length or content tangency while greatly helping some readers, and since then I've tried to do this as much as is reasonable, often adding such explanatory phrases on the second or third rewrite, as I don't always think about it the first time around. Other examples: Instead of "by an easy integration", say "letting u = ..." or "using integration by parts". Instead of "it is easy to show convergence", say "by limit comparison with a geometric series".

                          – Dave L Renfro
                          May 14 at 10:24













                          @DaveLRenfro Agreed, but at least "by easy integration" is better than easy to see. It at least signals to the reader that, awe I need to compute this integral to get to the next statement, the more specific the better assuming not many words are required. Absolutely agree.

                          – WetlabStudent
                          May 15 at 3:50





                          @DaveLRenfro Agreed, but at least "by easy integration" is better than easy to see. It at least signals to the reader that, awe I need to compute this integral to get to the next statement, the more specific the better assuming not many words are required. Absolutely agree.

                          – WetlabStudent
                          May 15 at 3:50











                          9














                          My personal opinion is that "it is easy to see" should really mean that it is easy for a reader who is the target audience to see, and can be used when that part of the proof is not an important piece, such as if it is a routine or tedious but uninteresting calculation. For example, it is better to write "it is easy to see that algorithm A runs in O(n^3) time" if it is just a bunch of for-loops that obviously take O(n^3) time, than to give a long and tedious proof just for the sake of formal rigour.



                          That said, it does happen that sometimes what one thinks is "easy to see" is not so easy for others to see, or perhaps even oneself after a few months of not looking at it. Hence having reviewers helps.






                          share|improve this answer




























                            9














                            My personal opinion is that "it is easy to see" should really mean that it is easy for a reader who is the target audience to see, and can be used when that part of the proof is not an important piece, such as if it is a routine or tedious but uninteresting calculation. For example, it is better to write "it is easy to see that algorithm A runs in O(n^3) time" if it is just a bunch of for-loops that obviously take O(n^3) time, than to give a long and tedious proof just for the sake of formal rigour.



                            That said, it does happen that sometimes what one thinks is "easy to see" is not so easy for others to see, or perhaps even oneself after a few months of not looking at it. Hence having reviewers helps.






                            share|improve this answer


























                              9












                              9








                              9







                              My personal opinion is that "it is easy to see" should really mean that it is easy for a reader who is the target audience to see, and can be used when that part of the proof is not an important piece, such as if it is a routine or tedious but uninteresting calculation. For example, it is better to write "it is easy to see that algorithm A runs in O(n^3) time" if it is just a bunch of for-loops that obviously take O(n^3) time, than to give a long and tedious proof just for the sake of formal rigour.



                              That said, it does happen that sometimes what one thinks is "easy to see" is not so easy for others to see, or perhaps even oneself after a few months of not looking at it. Hence having reviewers helps.






                              share|improve this answer













                              My personal opinion is that "it is easy to see" should really mean that it is easy for a reader who is the target audience to see, and can be used when that part of the proof is not an important piece, such as if it is a routine or tedious but uninteresting calculation. For example, it is better to write "it is easy to see that algorithm A runs in O(n^3) time" if it is just a bunch of for-loops that obviously take O(n^3) time, than to give a long and tedious proof just for the sake of formal rigour.



                              That said, it does happen that sometimes what one thinks is "easy to see" is not so easy for others to see, or perhaps even oneself after a few months of not looking at it. Hence having reviewers helps.







                              share|improve this answer












                              share|improve this answer



                              share|improve this answer










                              answered May 12 at 18:48









                              user21820user21820

                              74169




                              74169























                                  8














                                  Papers are not written with exhaustive detail. There is always a balance between explicitly stating your reasoning and sticking to the topic at hand. Sometimes there are points that don't take great insight to understand, and would be easy (though perhaps lengthy) for the reader to derive on their own. This is the first function of "it is easy to see".



                                  Another aspect is managing tone and audience expectation. Papers are written for an expert audience with the premise of pushing the bleeding edge of human knowledge. If the author of a paper states something obvious or well known, the reader may be confused: If it is emphasized in a modern paper, could it be that the author is not just a fool restating things everybody knows, but actually means to state something different? The competent reader could be confused by what the author meant to accomplish in stating something already well known or "easy to see". So the author preempts this, by acknowledging the point as something well-known and not novel, but mentioned for clarity and/or as a reminder, and not something to be taken as a novel or interesting claim. This is the second function of "it is easy to see".



                                  Some people like to criticize statements like "it can be shown" ("then why don't you show it?" - because it would be a distracting digression) or "it's obvious" ("then why say it?" - to show that the author is aware of it). They assume it is an exercise in arrogance on the part of the author. However, written communication is not just the text. There is also subtext and context. Well-structured text has a central point and a coherent tone. Going on every possible tangent, digressing into topics of wildly different level, does not necessarily serve these qualities. As such, phrases like "it is easy to see" can serve an important point in enabling effective communication of novel findings to an advanced, technical audience that has less time available to read than there are papers of interest.






                                  share|improve this answer




























                                    8














                                    Papers are not written with exhaustive detail. There is always a balance between explicitly stating your reasoning and sticking to the topic at hand. Sometimes there are points that don't take great insight to understand, and would be easy (though perhaps lengthy) for the reader to derive on their own. This is the first function of "it is easy to see".



                                    Another aspect is managing tone and audience expectation. Papers are written for an expert audience with the premise of pushing the bleeding edge of human knowledge. If the author of a paper states something obvious or well known, the reader may be confused: If it is emphasized in a modern paper, could it be that the author is not just a fool restating things everybody knows, but actually means to state something different? The competent reader could be confused by what the author meant to accomplish in stating something already well known or "easy to see". So the author preempts this, by acknowledging the point as something well-known and not novel, but mentioned for clarity and/or as a reminder, and not something to be taken as a novel or interesting claim. This is the second function of "it is easy to see".



                                    Some people like to criticize statements like "it can be shown" ("then why don't you show it?" - because it would be a distracting digression) or "it's obvious" ("then why say it?" - to show that the author is aware of it). They assume it is an exercise in arrogance on the part of the author. However, written communication is not just the text. There is also subtext and context. Well-structured text has a central point and a coherent tone. Going on every possible tangent, digressing into topics of wildly different level, does not necessarily serve these qualities. As such, phrases like "it is easy to see" can serve an important point in enabling effective communication of novel findings to an advanced, technical audience that has less time available to read than there are papers of interest.






                                    share|improve this answer


























                                      8












                                      8








                                      8







                                      Papers are not written with exhaustive detail. There is always a balance between explicitly stating your reasoning and sticking to the topic at hand. Sometimes there are points that don't take great insight to understand, and would be easy (though perhaps lengthy) for the reader to derive on their own. This is the first function of "it is easy to see".



                                      Another aspect is managing tone and audience expectation. Papers are written for an expert audience with the premise of pushing the bleeding edge of human knowledge. If the author of a paper states something obvious or well known, the reader may be confused: If it is emphasized in a modern paper, could it be that the author is not just a fool restating things everybody knows, but actually means to state something different? The competent reader could be confused by what the author meant to accomplish in stating something already well known or "easy to see". So the author preempts this, by acknowledging the point as something well-known and not novel, but mentioned for clarity and/or as a reminder, and not something to be taken as a novel or interesting claim. This is the second function of "it is easy to see".



                                      Some people like to criticize statements like "it can be shown" ("then why don't you show it?" - because it would be a distracting digression) or "it's obvious" ("then why say it?" - to show that the author is aware of it). They assume it is an exercise in arrogance on the part of the author. However, written communication is not just the text. There is also subtext and context. Well-structured text has a central point and a coherent tone. Going on every possible tangent, digressing into topics of wildly different level, does not necessarily serve these qualities. As such, phrases like "it is easy to see" can serve an important point in enabling effective communication of novel findings to an advanced, technical audience that has less time available to read than there are papers of interest.






                                      share|improve this answer













                                      Papers are not written with exhaustive detail. There is always a balance between explicitly stating your reasoning and sticking to the topic at hand. Sometimes there are points that don't take great insight to understand, and would be easy (though perhaps lengthy) for the reader to derive on their own. This is the first function of "it is easy to see".



                                      Another aspect is managing tone and audience expectation. Papers are written for an expert audience with the premise of pushing the bleeding edge of human knowledge. If the author of a paper states something obvious or well known, the reader may be confused: If it is emphasized in a modern paper, could it be that the author is not just a fool restating things everybody knows, but actually means to state something different? The competent reader could be confused by what the author meant to accomplish in stating something already well known or "easy to see". So the author preempts this, by acknowledging the point as something well-known and not novel, but mentioned for clarity and/or as a reminder, and not something to be taken as a novel or interesting claim. This is the second function of "it is easy to see".



                                      Some people like to criticize statements like "it can be shown" ("then why don't you show it?" - because it would be a distracting digression) or "it's obvious" ("then why say it?" - to show that the author is aware of it). They assume it is an exercise in arrogance on the part of the author. However, written communication is not just the text. There is also subtext and context. Well-structured text has a central point and a coherent tone. Going on every possible tangent, digressing into topics of wildly different level, does not necessarily serve these qualities. As such, phrases like "it is easy to see" can serve an important point in enabling effective communication of novel findings to an advanced, technical audience that has less time available to read than there are papers of interest.







                                      share|improve this answer












                                      share|improve this answer



                                      share|improve this answer










                                      answered May 12 at 18:09









                                      TruslyTrusly

                                      1,450112




                                      1,450112























                                          6














                                          Yes. It specifies a particular level of difficulty (not too easy and not too hard), thus managing the reader's expectations and directing their focus.



                                          The phrase will not be used for completely trivial deductions that can be done in half a second. If the previous sentence concluded that 2x=2, nobody would write "it is therefore easy to see that x=1". It's easy to see that it's easy to see, so there's no value in pointing out that it's easy to see.



                                          Likewise, if the deduction is difficult and requires hours to figure out, nobody would write "it's easy to see", because it's not, and saying it is will confuse the reader.



                                          It's the middle ground where the deduction can take a few seconds or maybe a few minutes, where the phrase is useful. A priori, the reader does not know the difficulty of the deduction - is this a half-second thing and I'm too dense to figure it out? Is this a difficult deduction and the author is remiss for neglecting a proof, or maybe he's just making stuff up?



                                          The statement "It's easy to see" signals that we're in the middle ground - No, you're not stupid for failing to recognize this immediately (if it were that easy, I wouldn't say anything). But yes, I'm confident that if you spend a minute you'll figure it out, so there's no need to encumber the paper with all the details.



                                          So much power in such a simple phrase.






                                          share|improve this answer






























                                            6














                                            Yes. It specifies a particular level of difficulty (not too easy and not too hard), thus managing the reader's expectations and directing their focus.



                                            The phrase will not be used for completely trivial deductions that can be done in half a second. If the previous sentence concluded that 2x=2, nobody would write "it is therefore easy to see that x=1". It's easy to see that it's easy to see, so there's no value in pointing out that it's easy to see.



                                            Likewise, if the deduction is difficult and requires hours to figure out, nobody would write "it's easy to see", because it's not, and saying it is will confuse the reader.



                                            It's the middle ground where the deduction can take a few seconds or maybe a few minutes, where the phrase is useful. A priori, the reader does not know the difficulty of the deduction - is this a half-second thing and I'm too dense to figure it out? Is this a difficult deduction and the author is remiss for neglecting a proof, or maybe he's just making stuff up?



                                            The statement "It's easy to see" signals that we're in the middle ground - No, you're not stupid for failing to recognize this immediately (if it were that easy, I wouldn't say anything). But yes, I'm confident that if you spend a minute you'll figure it out, so there's no need to encumber the paper with all the details.



                                            So much power in such a simple phrase.






                                            share|improve this answer




























                                              6












                                              6








                                              6







                                              Yes. It specifies a particular level of difficulty (not too easy and not too hard), thus managing the reader's expectations and directing their focus.



                                              The phrase will not be used for completely trivial deductions that can be done in half a second. If the previous sentence concluded that 2x=2, nobody would write "it is therefore easy to see that x=1". It's easy to see that it's easy to see, so there's no value in pointing out that it's easy to see.



                                              Likewise, if the deduction is difficult and requires hours to figure out, nobody would write "it's easy to see", because it's not, and saying it is will confuse the reader.



                                              It's the middle ground where the deduction can take a few seconds or maybe a few minutes, where the phrase is useful. A priori, the reader does not know the difficulty of the deduction - is this a half-second thing and I'm too dense to figure it out? Is this a difficult deduction and the author is remiss for neglecting a proof, or maybe he's just making stuff up?



                                              The statement "It's easy to see" signals that we're in the middle ground - No, you're not stupid for failing to recognize this immediately (if it were that easy, I wouldn't say anything). But yes, I'm confident that if you spend a minute you'll figure it out, so there's no need to encumber the paper with all the details.



                                              So much power in such a simple phrase.






                                              share|improve this answer















                                              Yes. It specifies a particular level of difficulty (not too easy and not too hard), thus managing the reader's expectations and directing their focus.



                                              The phrase will not be used for completely trivial deductions that can be done in half a second. If the previous sentence concluded that 2x=2, nobody would write "it is therefore easy to see that x=1". It's easy to see that it's easy to see, so there's no value in pointing out that it's easy to see.



                                              Likewise, if the deduction is difficult and requires hours to figure out, nobody would write "it's easy to see", because it's not, and saying it is will confuse the reader.



                                              It's the middle ground where the deduction can take a few seconds or maybe a few minutes, where the phrase is useful. A priori, the reader does not know the difficulty of the deduction - is this a half-second thing and I'm too dense to figure it out? Is this a difficult deduction and the author is remiss for neglecting a proof, or maybe he's just making stuff up?



                                              The statement "It's easy to see" signals that we're in the middle ground - No, you're not stupid for failing to recognize this immediately (if it were that easy, I wouldn't say anything). But yes, I'm confident that if you spend a minute you'll figure it out, so there's no need to encumber the paper with all the details.



                                              So much power in such a simple phrase.







                                              share|improve this answer














                                              share|improve this answer



                                              share|improve this answer








                                              edited May 13 at 16:35

























                                              answered May 13 at 14:30









                                              Meni RosenfeldMeni Rosenfeld

                                              26014




                                              26014























                                                  6














                                                  The author most often writes this to justify that he does not include a proof, that you can do yourself in a reasonable time. This assumes that you have the basic knowlege that is required in your field to understand the average paper.



                                                  If you think you're missing some parts and cannot see why it is easy, you usually can look up the "easy to see" (after eight pages of calculations) proofs in other manuscripts, text books or in the internet.



                                                  If they would include this proofs, the reviewers and editors would ask them, why they proof something, that is known from text books or "easy" to see. A paper should contribute something new and should not (and cannot) proof any details that it builds up on.



                                                  For more stuff that is not well-known or more complicated, the authors should provide a citation and write "the derivation of the formula and the proof can be found in [42]".



                                                  But when you cite a text book, then people can ask why do you cite text book A and not text book B? The correct citation would be the original paper, but it will usually be much harder to understand the topic in the original paper than in a text book.



                                                  And there are of course some hand-waving, when the author knows something to be true, but would need more time to proof it himself and does not consider it to be worth the effort.






                                                  share|improve this answer






























                                                    6














                                                    The author most often writes this to justify that he does not include a proof, that you can do yourself in a reasonable time. This assumes that you have the basic knowlege that is required in your field to understand the average paper.



                                                    If you think you're missing some parts and cannot see why it is easy, you usually can look up the "easy to see" (after eight pages of calculations) proofs in other manuscripts, text books or in the internet.



                                                    If they would include this proofs, the reviewers and editors would ask them, why they proof something, that is known from text books or "easy" to see. A paper should contribute something new and should not (and cannot) proof any details that it builds up on.



                                                    For more stuff that is not well-known or more complicated, the authors should provide a citation and write "the derivation of the formula and the proof can be found in [42]".



                                                    But when you cite a text book, then people can ask why do you cite text book A and not text book B? The correct citation would be the original paper, but it will usually be much harder to understand the topic in the original paper than in a text book.



                                                    And there are of course some hand-waving, when the author knows something to be true, but would need more time to proof it himself and does not consider it to be worth the effort.






                                                    share|improve this answer




























                                                      6












                                                      6








                                                      6







                                                      The author most often writes this to justify that he does not include a proof, that you can do yourself in a reasonable time. This assumes that you have the basic knowlege that is required in your field to understand the average paper.



                                                      If you think you're missing some parts and cannot see why it is easy, you usually can look up the "easy to see" (after eight pages of calculations) proofs in other manuscripts, text books or in the internet.



                                                      If they would include this proofs, the reviewers and editors would ask them, why they proof something, that is known from text books or "easy" to see. A paper should contribute something new and should not (and cannot) proof any details that it builds up on.



                                                      For more stuff that is not well-known or more complicated, the authors should provide a citation and write "the derivation of the formula and the proof can be found in [42]".



                                                      But when you cite a text book, then people can ask why do you cite text book A and not text book B? The correct citation would be the original paper, but it will usually be much harder to understand the topic in the original paper than in a text book.



                                                      And there are of course some hand-waving, when the author knows something to be true, but would need more time to proof it himself and does not consider it to be worth the effort.






                                                      share|improve this answer















                                                      The author most often writes this to justify that he does not include a proof, that you can do yourself in a reasonable time. This assumes that you have the basic knowlege that is required in your field to understand the average paper.



                                                      If you think you're missing some parts and cannot see why it is easy, you usually can look up the "easy to see" (after eight pages of calculations) proofs in other manuscripts, text books or in the internet.



                                                      If they would include this proofs, the reviewers and editors would ask them, why they proof something, that is known from text books or "easy" to see. A paper should contribute something new and should not (and cannot) proof any details that it builds up on.



                                                      For more stuff that is not well-known or more complicated, the authors should provide a citation and write "the derivation of the formula and the proof can be found in [42]".



                                                      But when you cite a text book, then people can ask why do you cite text book A and not text book B? The correct citation would be the original paper, but it will usually be much harder to understand the topic in the original paper than in a text book.



                                                      And there are of course some hand-waving, when the author knows something to be true, but would need more time to proof it himself and does not consider it to be worth the effort.







                                                      share|improve this answer














                                                      share|improve this answer



                                                      share|improve this answer








                                                      edited May 14 at 8:12

























                                                      answered May 12 at 13:47









                                                      alloallo

                                                      2,246518




                                                      2,246518























                                                          1















                                                          "You'll notice that ...."




                                                          First person always reads smoother, commands attention, and prevents overwtiting.



                                                          That said, you'll want to be clear who your audience is at the outset of the publication. State it explicitly who should be reading the text, and the assumed knowledge level of the subject matter.






                                                          share|improve this answer




























                                                            1















                                                            "You'll notice that ...."




                                                            First person always reads smoother, commands attention, and prevents overwtiting.



                                                            That said, you'll want to be clear who your audience is at the outset of the publication. State it explicitly who should be reading the text, and the assumed knowledge level of the subject matter.






                                                            share|improve this answer


























                                                              1












                                                              1








                                                              1








                                                              "You'll notice that ...."




                                                              First person always reads smoother, commands attention, and prevents overwtiting.



                                                              That said, you'll want to be clear who your audience is at the outset of the publication. State it explicitly who should be reading the text, and the assumed knowledge level of the subject matter.






                                                              share|improve this answer














                                                              "You'll notice that ...."




                                                              First person always reads smoother, commands attention, and prevents overwtiting.



                                                              That said, you'll want to be clear who your audience is at the outset of the publication. State it explicitly who should be reading the text, and the assumed knowledge level of the subject matter.







                                                              share|improve this answer












                                                              share|improve this answer



                                                              share|improve this answer










                                                              answered May 12 at 20:55









                                                              Carl V. LewisCarl V. Lewis

                                                              1662




                                                              1662























                                                                  0














                                                                  Yes, though it is sometimes abused.



                                                                  It's an assertion that if you think something is true you've probably got it right. It also sets the tone of what you're about to have to think through. It's common in other walks of life the other way round: if you have something that you are asserting that is non-trivial and is going to take some working though, you give your audience a warning.



                                                                  In maths, perhaps arrogantly (though unfortunately in my experience accurately), it is assumed that the statements in papers, talks, books, etc are involved. Thus instead of caveating each sentence with "this is hard", the few that are easy are caveated instead (simply as a way to reduce verbosity).



                                                                  The problem comes when the caveat is incorrect or is only correct after something highly non-trivial has become internalised.
                                                                  In this case it alienates those readers. As I imagine you can guess it not only hampers progress, it can also be seen as a bit of a middle finger. "If you don't get this you must not be cut out to be doing this sort of thing". Perhaps in some cases this is how its meant or just laziness. However its not always easy to know what will help your readers most and "nothing deep happens here" is far more likely.



                                                                  I don't like using "trivial" or "clear" because of the potential to be misread, but I have often wanted to express that sentiment.






                                                                  share|improve this answer




























                                                                    0














                                                                    Yes, though it is sometimes abused.



                                                                    It's an assertion that if you think something is true you've probably got it right. It also sets the tone of what you're about to have to think through. It's common in other walks of life the other way round: if you have something that you are asserting that is non-trivial and is going to take some working though, you give your audience a warning.



                                                                    In maths, perhaps arrogantly (though unfortunately in my experience accurately), it is assumed that the statements in papers, talks, books, etc are involved. Thus instead of caveating each sentence with "this is hard", the few that are easy are caveated instead (simply as a way to reduce verbosity).



                                                                    The problem comes when the caveat is incorrect or is only correct after something highly non-trivial has become internalised.
                                                                    In this case it alienates those readers. As I imagine you can guess it not only hampers progress, it can also be seen as a bit of a middle finger. "If you don't get this you must not be cut out to be doing this sort of thing". Perhaps in some cases this is how its meant or just laziness. However its not always easy to know what will help your readers most and "nothing deep happens here" is far more likely.



                                                                    I don't like using "trivial" or "clear" because of the potential to be misread, but I have often wanted to express that sentiment.






                                                                    share|improve this answer


























                                                                      0












                                                                      0








                                                                      0







                                                                      Yes, though it is sometimes abused.



                                                                      It's an assertion that if you think something is true you've probably got it right. It also sets the tone of what you're about to have to think through. It's common in other walks of life the other way round: if you have something that you are asserting that is non-trivial and is going to take some working though, you give your audience a warning.



                                                                      In maths, perhaps arrogantly (though unfortunately in my experience accurately), it is assumed that the statements in papers, talks, books, etc are involved. Thus instead of caveating each sentence with "this is hard", the few that are easy are caveated instead (simply as a way to reduce verbosity).



                                                                      The problem comes when the caveat is incorrect or is only correct after something highly non-trivial has become internalised.
                                                                      In this case it alienates those readers. As I imagine you can guess it not only hampers progress, it can also be seen as a bit of a middle finger. "If you don't get this you must not be cut out to be doing this sort of thing". Perhaps in some cases this is how its meant or just laziness. However its not always easy to know what will help your readers most and "nothing deep happens here" is far more likely.



                                                                      I don't like using "trivial" or "clear" because of the potential to be misread, but I have often wanted to express that sentiment.






                                                                      share|improve this answer













                                                                      Yes, though it is sometimes abused.



                                                                      It's an assertion that if you think something is true you've probably got it right. It also sets the tone of what you're about to have to think through. It's common in other walks of life the other way round: if you have something that you are asserting that is non-trivial and is going to take some working though, you give your audience a warning.



                                                                      In maths, perhaps arrogantly (though unfortunately in my experience accurately), it is assumed that the statements in papers, talks, books, etc are involved. Thus instead of caveating each sentence with "this is hard", the few that are easy are caveated instead (simply as a way to reduce verbosity).



                                                                      The problem comes when the caveat is incorrect or is only correct after something highly non-trivial has become internalised.
                                                                      In this case it alienates those readers. As I imagine you can guess it not only hampers progress, it can also be seen as a bit of a middle finger. "If you don't get this you must not be cut out to be doing this sort of thing". Perhaps in some cases this is how its meant or just laziness. However its not always easy to know what will help your readers most and "nothing deep happens here" is far more likely.



                                                                      I don't like using "trivial" or "clear" because of the potential to be misread, but I have often wanted to express that sentiment.







                                                                      share|improve this answer












                                                                      share|improve this answer



                                                                      share|improve this answer










                                                                      answered May 13 at 12:50









                                                                      ANoneANone

                                                                      1904




                                                                      1904























                                                                          0














                                                                          Yes, there are good reasons for the general meaning of phrases like this, but "It is easy to see that..." is a very poor choice of phrase for this meaning. Others have already suggested better phrases.



                                                                          As Buffy expresses, it's not expected for advanced papers to reference, let alone prove every conclusion used. The problem here is the passive voice, and, more importantly:



                                                                          It is easy to see. What is easy to see?



                                                                          I see two general phrase choices:




                                                                          1. Longer. If you want sentence flow to present a long thought, "From Y, we observe that with/from [few steps or concepts], X.", which carries the meaning more explicitly.


                                                                          2. Shorter. What is being said is, "Y. Y => X." (Y being the conclusion the reader is expected to know, from the audience the text is written for.)



                                                                          This way, you are at least specifying the subject of your sentence.



                                                                          Now, if Y is something you learn in high school, you would look silly specifying it; you would look as if you're proud that you can still do high school math. You write papers for your peers.



                                                                          It's a safe bet that one or more of your peers won't agree Y is obvious, but perhaps will say nothing in review to not "look dumb", when in fact s/he is your peer, just working in a different field.



                                                                          Widening the audience slightly from that seems a reasonable place to be.






                                                                          share|improve this answer






























                                                                            0














                                                                            Yes, there are good reasons for the general meaning of phrases like this, but "It is easy to see that..." is a very poor choice of phrase for this meaning. Others have already suggested better phrases.



                                                                            As Buffy expresses, it's not expected for advanced papers to reference, let alone prove every conclusion used. The problem here is the passive voice, and, more importantly:



                                                                            It is easy to see. What is easy to see?



                                                                            I see two general phrase choices:




                                                                            1. Longer. If you want sentence flow to present a long thought, "From Y, we observe that with/from [few steps or concepts], X.", which carries the meaning more explicitly.


                                                                            2. Shorter. What is being said is, "Y. Y => X." (Y being the conclusion the reader is expected to know, from the audience the text is written for.)



                                                                            This way, you are at least specifying the subject of your sentence.



                                                                            Now, if Y is something you learn in high school, you would look silly specifying it; you would look as if you're proud that you can still do high school math. You write papers for your peers.



                                                                            It's a safe bet that one or more of your peers won't agree Y is obvious, but perhaps will say nothing in review to not "look dumb", when in fact s/he is your peer, just working in a different field.



                                                                            Widening the audience slightly from that seems a reasonable place to be.






                                                                            share|improve this answer




























                                                                              0












                                                                              0








                                                                              0







                                                                              Yes, there are good reasons for the general meaning of phrases like this, but "It is easy to see that..." is a very poor choice of phrase for this meaning. Others have already suggested better phrases.



                                                                              As Buffy expresses, it's not expected for advanced papers to reference, let alone prove every conclusion used. The problem here is the passive voice, and, more importantly:



                                                                              It is easy to see. What is easy to see?



                                                                              I see two general phrase choices:




                                                                              1. Longer. If you want sentence flow to present a long thought, "From Y, we observe that with/from [few steps or concepts], X.", which carries the meaning more explicitly.


                                                                              2. Shorter. What is being said is, "Y. Y => X." (Y being the conclusion the reader is expected to know, from the audience the text is written for.)



                                                                              This way, you are at least specifying the subject of your sentence.



                                                                              Now, if Y is something you learn in high school, you would look silly specifying it; you would look as if you're proud that you can still do high school math. You write papers for your peers.



                                                                              It's a safe bet that one or more of your peers won't agree Y is obvious, but perhaps will say nothing in review to not "look dumb", when in fact s/he is your peer, just working in a different field.



                                                                              Widening the audience slightly from that seems a reasonable place to be.






                                                                              share|improve this answer















                                                                              Yes, there are good reasons for the general meaning of phrases like this, but "It is easy to see that..." is a very poor choice of phrase for this meaning. Others have already suggested better phrases.



                                                                              As Buffy expresses, it's not expected for advanced papers to reference, let alone prove every conclusion used. The problem here is the passive voice, and, more importantly:



                                                                              It is easy to see. What is easy to see?



                                                                              I see two general phrase choices:




                                                                              1. Longer. If you want sentence flow to present a long thought, "From Y, we observe that with/from [few steps or concepts], X.", which carries the meaning more explicitly.


                                                                              2. Shorter. What is being said is, "Y. Y => X." (Y being the conclusion the reader is expected to know, from the audience the text is written for.)



                                                                              This way, you are at least specifying the subject of your sentence.



                                                                              Now, if Y is something you learn in high school, you would look silly specifying it; you would look as if you're proud that you can still do high school math. You write papers for your peers.



                                                                              It's a safe bet that one or more of your peers won't agree Y is obvious, but perhaps will say nothing in review to not "look dumb", when in fact s/he is your peer, just working in a different field.



                                                                              Widening the audience slightly from that seems a reasonable place to be.







                                                                              share|improve this answer














                                                                              share|improve this answer



                                                                              share|improve this answer








                                                                              edited May 17 at 11:45

























                                                                              answered May 17 at 10:21









                                                                              Henrik ErlandssonHenrik Erlandsson

                                                                              1013




                                                                              1013















                                                                                  Popular posts from this blog

                                                                                  He _____ here since 1970 . Answer needed [closed]What does “since he was so high” mean?Meaning of “catch birds for”?How do I ensure “since” takes the meaning I want?“Who cares here” meaningWhat does “right round toward” mean?the time tense (had now been detected)What does the phrase “ring around the roses” mean here?Correct usage of “visited upon”Meaning of “foiled rail sabotage bid”It was the third time I had gone to Rome or It is the third time I had been to Rome

                                                                                  Bunad

                                                                                  Færeyskur hestur Heimild | Tengill | Tilvísanir | LeiðsagnarvalRossið - síða um færeyska hrossið á færeyskuGott ár hjá færeyska hestinum